<<

QUESTIONS

QUESTIONS AND ANSWERS: These are meant to be complimentary to the text. References to text are given, or ifthe learning point is contained within the question itself, that page is cited.

Patient 1: (Questions 1 through 4)

A 23-year-old female is brought to the emergency room after complaining of a severe headache (the worst in her life) and collapsing. Exam reveals stupor, BP 230/110, T 97, P 100, R 26, meningismus, subhyaloid hemorrhage on fundoscopic examination, systolic murmur over anterior chest becoming continuous over the back, delayed femoral pulsations and wide carrying angle of the arms (You notice this and confirm with family).

1. The single best explanation for her acute event is:

A. migraine headache B. thrombotic eVA c. endocarditis D. subarachnoid hemorrhage E. hypertensive encephalopathy

2. The best explanation ofher murmurs is:

A. aortic stenosis and ins~fficiency B. mitral stenosis and insufficiency C. atrial septal defect D. ventricular septal defect E. coarctation ofthe aorta QUESTIONS

3. The best explanation anatomically for her acute event is:

A. aneurysm of the circle ofWillis with bleeding into subarachnoid space B. vegetation on the aortic valve C. clot in the middle cerebral artery D. vasoconstriction E. neuronal spreading depression

4. Her congenital condition which ties the clinical picture together is:

A. Down syndrome B. Apert's syndrome C. Turner's syndrome D. Polyglandular endocrinopathy E. Neurofibromatosis

Patient 2: (Questions 5 and 6)

A 79-year-old white male presents with witnessed syncope (no seizure activity) and dyspnea. Exam reveals BP 180/100, P 60's with pauses, T 98, R 20; systolic scratchy harsh murmur over precordium radiating to sternal notch and carotids; displaced apical impulse, rales bilaterally. Monitor shows occasional two-second pauses; EKG shows left ventricular hypertrophy and complete heart block.

5. The next best test to work up this patient's problem is:

A. EEG B. Echocardiogram C. CT brain scan D. Right heart catheter E. Pulmonary functions

308 QUESTIONS

6. You find something surgically correctable on this test. In discussing the need for surgery with this patient you need to understand:

A. Surgery is for cosmetic reasons only B. Surgery will improve quality oflife, not quantity C. Surgery is indicated only for young patients D. Surgery is associated with a 50% mortality E. Untreated surgically, the patient has a 50% two year survival

Patient 3: (Questions 7 and 8)

A 50-year-old male patient complains in April of choking sensations at night which awaken him and force him to sit upright while sleeping. He has smoked for 40 years and his 67-year-old mother died recently of a cerebrovascular accident secondary to hypertension. Exam reveals BP 190/90, P 100, weight 278, few basilar rales, laterally displaced point of maximal impulse, frequent ectopic beats, 2+ pitting edema. EKG reveals left ventricular hypertrophy and bilateral atrial enlargement. Chest X-ray shows mild cardiac enlargement and fluid in fissures. While discussing these abnormal findings, you witness one of these episodes while the patient is sitting down.

7. The best explanation for the patient's symptoms is:

A. Allergic rhinitis B. Exacerbation of chronic obstructive pulmonary disease C. Sleep apnea syndrome D. Intermittent left ventricular dysfunction E. Reflux esophagitis

309 QUESTIONS

8. A physical exam finding to confirm your diagnosis is:

A. pale engorged nasal mucosa B. use ofaccessory muscles ofrespiration C. excessive oropharyngeal tissue D. S3 gallop E. Air swallowing and hiccups

Patient 4: (Questions 9 and 10)

A 25-year-old Hispanic male from a large Eastern US city is admitted with fever and malaise. On exam you note a temperature of 103 degrees, palatal and conjunctival petechiae, a systolic grade IIINI murmur along left sternal border, tenderness and fullness of LUQ, and track marks with cellulitis over antecubital fossae and arms.

9. You order the appropriate tests which include all EXCEPT:

A. blood cultures B. chest X-ray C. complete blood count D. biopsy E. echocardiogram

10. After he arrives on the floor his blood pressure, which had been 120/80, becomes 140/50, his heart rate increases from 100 to 120 and he becomes pale and diaphoretic as he complains of chest pain. Auscultation ofhis heart reveals a new soft, short diastolic murmur, more prominent as he leans forward. What has happened?

A. Myocardial infarction B. Pericarditis C. Septic embolus to spleen D. Perforated aortic cusps E. Exacerbation ofrheumatic heart disease

310 QUESTIONS

Patient 5: (Questions 11 through 13)

A 45-year-old white female underwent cholecystectomy and on day two became acutely dyspneic after straining to pass flatus. She became tachypneic, tachycardic and diaphoretic. She had a few expiratory wheezes bilaterally. Arterial blood gases revealed pH 7.50 pC02 25, p02 60. Chest X-ray was unremarkable.

11. The most likely explanation for her acute problem is:

A. myocardial infarction B. pulmonary embolus C. post operative ileus D. sepsis from wound infection E. gallstone ileus

12. You manage the patient medically and she stabilizes. After several days she develops gangrenous areas over the lateral thighs. The most likely cause ofthis new problem is:

A. Embolization to and infarction ofskin from infective endocarditis B. Development ofnecrotizing fasciitis from a gangrenous gallbladder C. Pressure necrosis from bed rest D. Allergic reaction to a cephalosporin E. Coumadin skin necrosis

13. An underlying abnormal protein is searched for. You want to know her levels of:

A. Protein S B. Protein C C. Rheumatoid factor D. ANA E. Anti-thrombin ill

311 QUESTIONS

14. The most common cause ofsudden death is:

A. arrhythmia B. ruptured myocardium C. electromechanical dissociation D. massive pulmonary embolus

15. Placement ofthe Swan-Ganz catheter has the following potential complication (s):

A. ventricular ectopy B. right bundle branch block pattern C. perforation ofthe pulmonary artery D. pulmonary infarction E. all ofthe above

16. The most common side-effect ofnitroglycerin preparations is:

A. diarrhea B. pounding headache C. nausea D. chest pain

17. The patient has an irregularly irregular heart rate of 140 beats per minute, but his other vital signs are stable. The most probable dysrhythmia in this patient is:

A. ventricular tachycardia B. ventricular fibrillation C. atrial fibrillation D. sinus tachycardia

312 QUESTIONS

18. Which ofthe following valvular heart lesions produces a loud, snapping S1, an opening snap, and a diastolic rumble?

A. mitral stenosis B. mitral regurgitation C. aortic stenosis D. aortic regurgitation

19. Coxsackie B virus is the most common cause ofacute myocarditis in the United States, associated with fever, arrhythmias, chest pain, and possibly transient congestive .

___ True ---False

20. A common acute stress on the myocardium that precipitates congestive heart failure is:

A. acute volume or salt load B. ischemia or infarction C. uncontrolled hypertension D. hyperthyroidism E. all ofthe above

21. Uncommon causes ofhypertension include:

A. pheochromocytoma B. Conn's syndrome ofaldosterone secreting adrenal adenoma C. renal artery stenosis D. coarctation ofthe aorta E. all ofthe above

313 QUESTIONS

22. The drug ofchoice in a life-threatening hypertensive crisis is:

A. nitroprusside intravenously B. beta blockers C. nifedipine D. ACE inhibitors

23. The most common cause ofstroke in the US is:

A. hemorrhage B. arterial thrombosis C. cardiac source emboli D. trauma

24. Lacunar strokes are a family ofsyndromes that occur in patients with hypertension and atherosclerosis.

False ---True ---

25. The most probable etiology ofacute stroke in a patient with atrial fibrillation, mitral stenosis, dilated cardiomyopathy or ventricular aneurysm would be:

A. thrombotic B. embolic C. hemorrhagic

26. A transient ischemic attack (TIA) presents as an acute neurological deficit that must resolve (by definition) within:

A. one month B. seven days C. three days D. 24 hours

314 QUESTIONS

27. Approximately 25 percent of patients with TIAs suffer a cerebrovascular accident within five days and often within the first month ofthe first transient ischemic attack.

---True ---False

Patient 6: (Questions 28 - 32)

Mr. Green is a 50 year old male who presented with one hour of chest pressure and electrocardiographic findings of acute inferior myocardial infarction. He denied known heart disease, contraindications to thrombolytic therapy were not elicited, and he underwent successful thrombolysis.

28. Within hours, he complains ofright groin pain and develops an inguinal mass. On further questioning, he admits to being hospitalized for chest pain 48 hours prior to this admission. He is experiencing:

A. Dissection offemoral aneurysm B. Enlargement ofinguinal hernia C. Bleeding at site ofpreviously unknown femoral stick D. Infiltration ofintravenous fluids

29. You attend to this problem and the patient stablizes. The next day he worsens again with severe heart failure and you hear a new pansystolic murmur, grade IIIIVI, along the left sternal border with a parasternal thrill. You suspect:

A. Pulmonary embolus B. Rupture ofinterventricular septum C. Dressler's syndrome D. Pericardial tamponade

315 QUESTIONS

30. A rapid confinnation ofthe diagnosis is with:

A. 2D Echocardiogram and saline bubbles B. Response to pressor agents C. Emergency computed tomography scan with contrast D. Emergency heart catheterization E. scan

31. The latest problem is a recognized complication of:

A. myocardial infarction B late administration ofthrombolytic therapy C. chest trauma D. all ofthe above E. none ofthe above

32. Appropriate treatment might include:

A. emergent surgical repair and intra-aortic balloon support B. thrombectomy C. medical management D. prolonged bed rest E. inferior vena cava filter

Patient 7:

A 60 year old woman presents with dyspnea, dysphagia, weight loss, and an irregularly irregular rhythm with a rate of 165.

316 QUESTIONS

33. Likely underlying etiologies include all EXCEPT:

A. coronary artery disease B. hyperthyroidism C. atrial myxoma D. alcoholism E. pulmonary embolism

Patient 8: (Questions 34 - 36)

A high school basketball star was noted on the preseason physical examination to have an apical systolic murmur, the intensity of which decreased with squatting and hand grip. He was advised to have an echocardiogram but continued to practice. During a strenuous workout he collapsed and could not be resuscitated.

34. Autopsy revealed:

A. No pathology B. Pulmonary embolus C. Myocardial infarction D. Histologic cellular disarray ofmyocardium E. Bicuspid aortic valve

35. Which drug is to be avoided in the above condition?

A. Propranolol B. Thrombolytics C. Digitalis D. Adriamycin E. Capoten

317 QUESTIONS

36. What is the reason for sudden death in the above condition?

A. Arteriovenous shunting B. Brainstem infarction C. Ventricular arrhythmia D. Hypovolemic shock E. Lactic acidosis

Patient 9:

37. A 53-year-old male presented to the emergency room with a one-month history ofdyspnea on exertion and vague substernal chest discomfort. He has smoked two packs per day of cigarettes, has hypercholesterolemia, and a positive family history ofcoronary artery disease. His exam was unremarkable except for a loud pulmonic component ofthe second heart sound. The EKG showed nonspecific T wave changes and sinus tachycardia. Echo• cardiography revealed elevated right heart and pulmonary artery pressures. The patient died suddenly. Autopsy was performed which revealed:

A. Coronary thrombosis and acute anterior MI B. Coronary artery disease and massive pulmonary embolus C. Coronary artery disease and right ventricular hypertrophy D. Patient foramen ovale and paradoxical embolus E. Acute hemorrhagic pericarditis with tamponade

38. All ofthe following statements regarding thrombolytic therapy are true EXCEPT:

A. Streptokinase is most effective when given 6 - 24 hours after onset ofsymptoms of myocardial infarction B. Alteplase is effective when given 6-12 hours after onset ofmyocardial infarction C. 30% ofpatients are excluded from receiving thrombolytics because ofprolonged symptom duration at the time when initially seen D. The traditional time window for administration ofthrombolytics has been within six hours ofsymptoms E. In addition to myocardium salvage, the "open artery hypothesis" includes limitation of left ventricular dilatation and aneurysm formation, improved healing, better electrical stability, reduced mural thrombus formation, and potential collateral circulation to remote ischemic zones

318 QUESTIONS

39. In pulmonary function testing, a decreased FEV-l (forced expiratory volume in one second) is most indicative of

A. obstructive lung disease B. restrictive lung disease C. small airways disease D. sleep apnea

40. Hypoxemia derives largely from the following pulmonary disorder:

A. right-to-Ieft shunt B. alveolar hypoventilation C. diffusion abnormality D. all ofthe above

41. Hypoventilation can be caused by:

A. B. metabolic alkalosis C. sedatives D. E. all ofthe above

42. The most common pathogen found in the sputum ofbronchitics is:

A. Pneumococcus (Streptococcus pneumoniae) B. Hemophilus infJuenzae C. BranhameIIa (MoraxeIIa) catarrhalis D. all ofthe above

319 QUESTIONS

43. Important associations with asthmatics include:

A. aspirin sensitivity or allergy B. history ofnasal polyps C. sinusitis D. all ofthe above

44. The gold standard for the definitive diagnosis ofa pulmonary embolus is:

A. a VQ scan B. pulmonary angiography C. chest X-ray D. CT scan ofthe chest

45. The leading cause ofexudative pleural effusions is:

A. cancer B. congestive heart failure C. nephrotic syndrome D. bacterial infections

46. A common setting for aspiration pneumonia is:

A. depressed mental status B. nasogastric tube C. seIzure D. drunkenness E. all ofthe above

320 QUESTIONS

47. The differential diagnosis ofhemoptysis may include:

A. cancer B. bronchiectasis C. tuberculosis D. lung abscess E. pulmonary infarction F. all ofthe above

48. Cor pulmonale is a term applied to right ventricular enlargement secondary to pulmonary hypertensive lung disease or disorders ofthe thorax:

---True ___ False

49. An increased A-a gradient represents:

A. hypoxemia B. hypercapnia C. pneumorua D. shunting E. metabolic alkalosis

50. A 36-year-old white female presents with pleurisy, dyspnea, and ABGs showing hypoxemia and respiratory alkalosis. You want to ask about all EXCEPT:

A. use oforal contraceptives B. prior similar episodes C. traveler's diarrhea D. pregnancy E. cross country travel by auto

321 QUESTIONS

51. Asbestosis shows which one ofthe following on chest X-ray?

A. bilateral upper lobe fluffy infiltrates B. bilateral lower lobe fibrosis C. bilateral upper lobe fibrosis D. bilateral upper lobe cavities E. bilateral lower lobe cavities

52. Farmer's lung is the prototype of:

A. connective tissue lung disease B. hereditary lung disease C. nosocomial lung disease D. hypersensitivity lung disease E. infectious lung disease

53. All except one ofthe following are problematic in patients with severe COPD:

A. Nocardia B. Mycobacterium kansasii C. typical tuberculosis D. mv E. Mycobacterium avium complex

54. The treatment ofchoice for allergic asthma is:

A. oral beta-2 bronchodilators B. oral theophylline C. inhaled steroids D. oral steroids E. parenteral steroids

322 QUESTIONS

55. Chronic bronchitics most often complain of:

A. chest pains, substernal B. chest pains, pleuritic C. cough and sputum D. air hunger E. weight loss

56. Patient with emphysema most often complain of:

A. chest pains, substernal B. fever and sputum C. cough and sputum D. dyspnea E. weight loss

57. Nicotine addiction is as powerful an addiction as heroin.

---True ---False

58. Indications for oxygen include all EXCEPT:

A. hypoxemia with p02 below 55 B. desaturation with exercise C. secondary polycythemia D. cor pulmonale E. FEV! below 1.9 liters

59. Erythromycin as initial outpatient therapy for community-acquired pneumonia is a good choice because ofits effectiveness against:

A. Pneumococcus B. Mycoplasma C. Legionnaires' disease D. all ofthe above

323 QUESTIONS

60. One judges the quality ofa sputum smear by:

A. numbers ofgram-positive bacteria per high-powered field B. numbers ofgram-negative bacteria C. presence ofeosinophilic staining mucus D. presence ofleukocytes and absence ofsquamous epithelial cells

61. The choice ofempiric antibiotic therapy in a 65-year-old male with an infiltrate depends upon all but:

A. presence ofunderlying COPD B. acquisition ofpneumonia in community or hospital C. presence ofunderlying lung abscess D. creatinine E. P02

Questions 62 - 64 involve Patient #10

A 55 year old white male veteran presented with a 3 day history oflethargy and confusion. His appetite has been poor and he had not had a bowel movement in 5 days. Physical examination revealed mild hypotension and profound generalized weakness. His low back and ribs were tender to palpation and movement.

Lab: Hgb 7.8 gldl, creat 4.0, calcium 16.2, SGOT 45, bilirubin 1.2

62. The most likely diagnosis is: (one best answer)

A. Hyperparathyroidism B. Multiple myeloma C. Milk alkali syndrome D. Hepatorenal syndrome E. Calcium oxalate kidney stone

324 QUESTIONS

63. The most urgent aspect ofhis treatment is institution of: (one best answer)

A. Antibiotics B. Antipyretics C. Fluids D. Antacids E. Steroids

64. The reason for his clinical presentation includes all EXCEPT:

A. Lack oforal intake B. Polyuria caused by hypercalcemia C. Confusion caused by hypercalcemia D. Constipation due to hypercalcemia E. Middle cerebral artery thrombosis

QUESTIONS 65 - 67 deal with Patient #11

An 18 year old white male presented to his physician in early September with tea colored urine and leg swelling. He had been previously healthy except for a sore throat several days earlier, which he had ignored due to rush week activities at his college campus. Several new friends also had been ill.

Exam revealed BP 180/116. P 90, T 98, R 26. Neck veins were slightly distended, throat was clear, apical impulse slightly displaced to L ofmid-clavicular line. clear, abdomen soft, 1-2+ pitting pretibial edema. Creatinine was 3.2, Hgb 11.9.

65. You want to personally examine his: (Your attending will say all ofthese, but choose one)

A. Blood smear B. Sputum C. Freshly voided urine D. Serum protein electrophoresis E. Chemistry profile

325 QUESTIONS

66. The best explanation for his findings is:

A. Stress due to leaving home B. Chlamydia pharyngitis sequelae C. Mycoplasma pharyngitis sequelae D. Acute post-streptococcal glomerulonephritis E. Coxsackie B myocarditis post-pharyngitis

67. His prognosis is (one best answer):

A. Poor because ofrenal failure B. Good because he contracted his illness in an"epidemic" rather than "sporadic" form C. Poor because offluid overload D. Good because he sought medical care promptly E. Poor because he did not seek medical care promptly

QUESTIONS 68 - 70 deal with Patient #12

A 49 year old mildly obese alcoholic white male executive presented to the ED with L flank pain radiating to the groin which was intermittent, colicky, and severe. Two years earlier he had been seen in the same ED for an acutely swollen, hot, painful R great toe but did not seek ongoing care after emergency treatment.

68. All ofthe following are important to the diagnosis ofhis problem EXCEPT:

A. KUB X-ray (kidneys, ureter, bladder) B. Urinalysis C. Chest X-ray D. Foot X-ray E. Intravenous pyelogram

326 QUESTIONS

69. His most probable metabolic problem deals with: (choose one)

A. Calcium B. Glucose C. Alcohol D. Lipids E. Uric acid

70. Treatment might include all EXCEPT:

A. Dietary modification B. High fluid intake C. Benemid D. Allopurinol E. Toradol

71. The number one cause ofhyponatremia in hospitalized patients is:

A. syndrome ofinappropriate antidiuretic hormone (SIADH) B. congestive heart failure C. dehydration D. infusion ofhypotonic solutions

72. Laboratory values showing a BUN of50, a creatinine of6, a urinary sodium of60, a urine osmolality of200, and a urine sediment showing renal epithelial cells is more indicative of:

A. acute tubular necrosis B. prerenal azotemia

327 QUESTIONS

73. The cause(s) ofpapillary necrosis ofthe kidneys include all EXCEPT:

A. analgesic abuse B. C. sickle cell anemia D. pyelonephritis E. polycystic kidney

74. Struvite renal stones (comprised ofmagnesium, ammonia and phosphate) are caused by urea-splitting bacteria and give a characteristic:

A. low urine pH B. high urine pH

75. The differential diagnosis ofbilaterally large kidneys includes all EXCEPT:

A. Myeloma B. Lymphoma C. End stage renal disease D. Hydronephrosis E. Polycystic disease F. Amyloid

76. Prevention ofglomerular hypertension in diabetes and delay ofnephropathy is thought to occur with which class ofdrugs?

A. ACE inhibitors B. Beta blockers C. Calcium channel blockers D. Thiazides E. Alpha blockers

328 QUESTIONS

77. Use ofintravenous contrast agents may precipitate renal failure in all except which group?

A. Mutiple myeloma B. Diabetics C. Dehydration D. Hodgkins disease E. Pre-existing renal insufficiency

78. Hyperkalemia in a healthy person on no drugs is usually due to:

A. Addison's disease, occult B. Use ofsalt substitute C. Rhabdomyolsis D. ofspecimen E. Vitamin and mineral supplements

79. The most important determinant ofseizure activity secondary to hyponatremia is:

A. Absolute value ofserum sodium B. Value ofserum sodium relative to creatinine C. Duration ofhyponatremia D. Presence offever E. Rapidity ofdecline in serum sodium

80. Correction ofhyponatremia must be done slowly to prevent which irreversible complication?

A. Precipitation ofcongestive heart failure B. Pontine myelinolysis C. Transient ischemic attack D. Hemolysis E. Rhabdomyolysis

329 QUESTIONS

81. The group most vulnerable to nonsteroidal agents is:

A. Elderly men B. Elderly women C. Adolescents D. Children E. Infants

82. Indications for hemodialysis include all EXCEPT:

A. End stage renal failure B. Volume overload unresponsive to medical management C. Hyperkalemia unresponsive to medical management D. Acidosis unresponsive to medical management E. Acute acetaminophen overdose

83. The anemia ofchronic renal failure is physiologically less severe than indicated by the level ofhemoglobin due to all EXCEPT:

A. The patient is less active and metabolic demands are less B. The onset is gradual, allowing for adaptation C. Acidosis shifts the 2-3 DPG curve to the right D. Acidosis shifts the 2-3 DPG curve to the left

84. Which ofthe hereditary forms ofrenal failure is characterized by a relatively normal hemogloblin?

A. Alport's syndrome B. Fabry's disease C. Medullary sponge kidney D. Fanconi syndrome E. Polycystic kidney disease

330 QUESTIONS

85. Which clinical findings and what renal lesion are associated with occasional recreational paint sniffing (toluene poisenir..g)?

A. Bone disease (osteomalacia in adults), autosomal dominant inheritance, distal type 1 renal tubular acidosis B. Renal and ureteral stones due to cystinuria C. Proximal tubular wasting ofelectrolytes and amino acids known as Fanconi syndrome D. Bicarbonate wasting, acidosis, severe hypokalemia with muscle weakness, classic distal type 1 renal tubular acidosis

86. Chronic urinary tract infections are associated with which type ofrenal stone?

A. Struvite B. Calcium C. Uric acid D. Calcium and uric acid E. Oxalate

87. Intestinal hyperoxaluria such as seen in terminal ileal disease results in the formation ofwhich kinds ofrenal stones?

A. Struvite B. Calcium C. Uric acid D. Calcium and uric acid E. Oxalate

88. The prophylactic treatment for stones formed in diseases resulting in intestinal hyperoxaluria is:

A. Thiazide diuretics B. Allopurinol C. Antibiotics D. Surgery E. Supplemental calcium, cholestyramine

331 QUESTIONS

89. Diabetics who also take nonsteroidal antiinflammatory drugs are prone to hyperkalemia and low urinary pH due to:

A. Classic distal type I renal tubular acidosis B. Proximal type II renal tubular acidosis C. Bicarbonate wasting D. Generalized distal type IV renal tubular acidosis with decreased aldosterone effect E. A combination ofall ofthe above

90. Alcoholic ketoacidosis is a disorder due to:

A. Underlying diabetes mellitus B. Pancreatic insufficiency due to repeated bouts ofpancreatitis C. Accidental ingestion ofisopropyl alcohol D. Accidental ingestion ofmethanol E. Binge drinking, starvation, and hepatic deplection ofglycogen

91. Bartter's syndrome is characterized by all EXCEPT:

A. Hypokalemia B. Hypertension C. Excess renin D. Excess aldosterone E. Resembles diuretic use/abuse F. Inability to resorb chloride in loop ofHenle

92. Low urinary chloride (less than 20 mEq/L) is seen in all ofthe following states EXCEPT:

A. Chronic vomiting B. Laxative abuse C. Chronic diarrhea D. Diuretic use

332 QUESTIONS

93. Metabolic alkalosis accompanies all ofthe following syndromes except which?

A. Prolonged nasogastric suctioning B. Bartter's syndrome C. Overdiuresis D. Ventilator dependence in C02-retaining chronic lung patient E. Renal failure

94. Causes ofthe syndrome ofinappropriate ADH include all but:

A. Small cell lung cancer B. Sarcoid involvement ofhypothalamic-pituitary axis C. Pain D. Narcotics E. Vincristine F. Cyclophosphamide G. Brain injury

95. The dreaded complication resulting from overzealous correction ofhypematremia is:

A. Seizures B. Confusion C. Congestive heart failure D. Pulmonary embolism E. Pontine myelinolysis

96. Magnesium-wasting drugs include all except:

A. Most antacids B. Digoxin C. Ticarcillin D. Most diuretics E. Amphotericin B F. Pentamidine G. Cis-platinum

333 QUESTIONS

97. The anemia ofchronic renal failure is multifactorial and may be due to all ofthe following except which?

A. Decreased production oferythropoietin due to decreased nephron size B. Hemolysis C. Fibrosis ofthe marrow due to hyperparathyroidism D. Blood loss through the GI tract and dialysis E. Inadequate replacement offolic acid F. Aluminum intoxication G. Hypersplenism H. Aplastic anemia

98. Decreased need for red cell transfusion and the complications thereofhas been accomplished through the use ofwhich drug in end-stage renal patients?

A. Folic acid B. Iron replacement C. Erythropoietin D. Interferon 2-alpha E. Pyridoxine

99. Erythema migrans, fever, chills, migratory polyarthritis, neurologic, and cardiac disease are all findings in:

A. Lyme disease B. gout C. septic arthritis D. pseudogout E. polyarteritis nodosa

334 QUESTIONS

100. Symmetric arthritis, morning stifthess, cervical spine involvement, and development ofsicca syndrome is most compatible with:

A. rheumatoid arthritis B. SLE C. scleroderma D. degenerative joint disease E. polyarteritis nodosa

101. The seronegative spondyloarthropathies include all but:

A. ankylosing spondylitis B. Reiter's syndrome c. psoriatic arthritis D. systemic lupus erythematosus E. inflammatory bowel disease

102. Drug-induced lupus is associated with:

A. hydralazine B. procainamide C. phenytoin D. isoniazid E. all ofthe above

103. Necrotizing vasculitis, granulomas ofthe upper and lower respiratory tracts, and glomerulonephritis are manifestations of:

A. Wegener's granulomatosis B. polyarteritis nodosa C. Henoch-Schonlein purpura D. erythema nodosum E. all ofthe above

335 QUESTIONS

104. An elderly patient has shoulder girdle stiflhess and hip pain. His sedimentation rate is 85. The most likely diagnosis is: (one best answer)

A. rheumatoid arthritis B. lupus C. polymyalgia rheumatica D. polymyositis E. degenerative joint disease

105. Scleroderma has all ofthe following features EXCEPT:

A. telangiectasias B. diarrhea C. renal failure D. hypertension E. diabetes

106. Mortality from lupus is due to all but one ofthe following:

A. renal failure B. respiratory failure C. Libman-Sacks endocarditis D. CNS events

107. The diagnosis oflupus is made by a positive ANA.

---True ---False

336 QUESTIONS

108. Arthrocentesis is important in the evaluation ofmonoarticular arthritis primarily to rule out: (choose one)

A. rheumatoid arthritis B. lupus C. vasculitis D. septic joint E. trauma

109. Temporal arteritis can be a medical emergency ifuntreated due to irreversible:

A. renal failure B. blindness C. muscle weakness D. respiratory failure E. cardiomyopathy

110. Chylous effusions contain:

A. uric acid crystals B. calcium pyrophosphate C. triglycerides D. porphyrins

111. Hepatitis B is associated with some cases of:

A. polymyositis B. polymyalgia rheumatica C. polyarteritis nodosa D. Caplan's syndrome E. lymphangioleiomyomatosis

337 QUESTIONS

112. The renal effects ofnonsteroidals are most pronounced in which population?

A. young women B. young men C. elderly women D. elderly men E. children

113. The gastrointestinal side effects ofnonsteroidals can be minimized by which one ofthe following?

A. administration with food B. use ofantacids C. use ofH2 blockers D. use ofsucralfate E. use ofmisoprostel F. all ofthe above

114. Which bleeding diathesis is associated with chronic polyarthritis?

A. Von Willebrand's B. Coumadin use C. factor 8 deficiency D. factor 13 deficiency E. autoimmune thrombocytopenia

115. Hazards ofarthrocentesis include all EXCEPT:

A. sudden lowering ofjoint fluid pressure B. introduction ofbacteria into joint space C. injecting corticosteroid into an infected joint D. needlestick injury to doctor E. corticosteroid-induced pain flare

338 QUESTIONS

116. Gottron's sign is pathognomonic of:

A. rheumatoid arthritis B. ankylosing spondylitis C. systemic lupus erythematosus D. discoid lupus E. dermatomyositis

117. Immunoglobulin staining ofthe dermal-epidermal junction in clinically normal skin is diagnostic of:

A. rheumatoid arthritis B. ankylosing spondylitis C. systemic lupus erythematosus D. discoid lupus E. dermatomyositis

118. All ofthe following are characteristic ofankylosing spondylitis EXCEPT:

A. low back pain begins before age 30 B. HLA-B27 antigen present> 90% C. 30° oblique views ofsacroiliac joints are helpful in showing sacroiliitis D. predominantly female sex E. "bamboo spine" decreases range ofmotion

119. Polyarteritis nodosa is associated with which infection?

A. Hepatitis A B. Hepatitis B C. Hepatitis due to cytomegalovirus D. Hepatitis due to Ebstein-Barr Virus

339 QUESTIONS

120. Long-term disability due to post-salmonella arthritis is predicted by:

A. immediate acute joint disease B. delayed acute joint disease C. multiple joint involvement D. failure to clear diarrhea E. HLA-B27 positivity

121. The best therapy for chronic low back pain due to degenerative disk disease and muscle spasm is:

A. laminectomy B. steroid injections in paravertebral musculature C. saline or sterile water injections in paravertebral musculature D. muscle strengthening exercises and reassurance E. Xylocaine injections offacet joints

122. Which drug should be avoided in the treatment ofacute gout?

A. naproxen B. indomethacin C. allopurinol D. colchicine E. benemid

340 QUESTIONS

123. Match the disease with the best antibody choice

A. Rheumatoid arthritis 1. ANA, antinucleolar pattern

B. Systemic lupus erythematosus 2. anti-RNP

C. Mixed connective tissue disease 3. anti-Jo

D. Polymyositis 4. antibody to single-stranded DNA, antihistone

E. Drug-induced lupus 5. anti-centromere antibody

F. Scleroderma 6. ANA, anti Smith

G. CREST syndrome 7. Anti SS-A, SS-B

H. Sjogren's 8. IgM reacting against IgG fragment

124. In a known penicillin-allergic patient, the incidence ofallergy to a cephalosporin is approximately

A. 10-15 percent B. 30 percent C. 50 percent D. 90 percent

125. The limiting factor in the use ofAmphotericin B is its

A. renal toxicity B. CNS toxicity C. phlebitis D. bone marrow suppression

341 QUESTIONS

126. Approximately two-thirds ofepisodes ofseptic shock are caused by gram-positive bacteria.

---True ---False

127. Match the meningitis type on the left with the best choice poplulation on the right:

A. Meningococcal 1. AIDS B. Pneumococcal 2. Military recruits C. Hemophilus 3. Basilar skull fracture D. Cryptococcal 4. Terminal complement component deficiency E. Recurrent gonococcal 5. Children

128. Reactivation ofinactive tuberculosis has historically occurred at the rate of:

A. 75% per year B. 50% per year C. 20% per year D. 12% per year E. 2% per year

129. Resurgence ofTB is a problem in this decade. The overall US incidence has increased by:

A. 5% B. 10% C. 18% D.27% E. 42%

342 QUESTIONS

130. All ofthe following are true about treatment with retroviral agents EXCEPT:

A. Combination therapy with nucleoside analogs is helpful B. AZT therapy should begin at the time ofconfirmation ofHIV positivity C. Retroviral therapy should be based on CD4 count D. Protease inhibitors and nucleoside analog combined therapy is useful in those failing nucleosides alone E. The goal oftherapy is to increase CD4 count

131. Lesions ofcondyloma lata are seen with which?

A. Primary syphilis B. Secondary syphilis C. Tertiary syphilis D. Latent syphilis E. Congenital syphilis

132. Patients with a long term known positive PPD and negative chest X-ray who are greater than 35 years ofage are not treated with isoniazid because:

A. risk ofhepatotoxicity is too great above this age B. too expensive C. outcome not altered D. there is no risk for recurrence

133. Roth spots, Janeway lesions, embolic glomerulophritis, and Osler's nodes are all terms applied to:

A. drug fever B. infectious endocarditis (bacterial endocarditis) C. pneumococcal pneumonia D. myocarditis

343 QUESTIONS

134.. Which ofthe following symptoms of urinary tract infections is most useful in pinpointing the source as being an upper versus a lower tract infection?

A. dysuria B. frequency C. fever D. flank pain

13 5. The number one cause ofcystitis in the outpatient population is:

A. E coli B. Pseudomonas C. Serratia D. Proteus

136. Brodie's abscess, sequestrum, and involucrum are terms used with:

A. osteomyelitis B. cellulitis C. erysipelas D. sepsis

137. A 20-year-old patient presents with fever, sore throat, enlarged tonsils, and bilateral posterior cervical denopathy. The differential diagnosis includes all but one ofthe following:

A. streptococcal pharyngitis B. chlamydial pharyngitis C. infectious mononucleosis D. congenital rubella syndrome E. nasopharyngeal carcinoma F. cytomegalovirus

344 QUESTIONS

138. The causative organism in Lyme disease is:

A. Treponema pal/idum B. Helicobacter C. Ixodides D. Borrelia burgdorferi E. Treponema (yaws)

139. A 50-year-old male oil executive presents with spiking fever and rigors. He has recently returned from a business trip to Kenya. To evaluate him, you do all ofthe following EXCEPT: (choose one)

A. ask about sexual intercourse with natives B. look at peripheral blood smear C. measure liver function tests D. palpate for splenomegaly E. check stools for ova and parasites F. order MRI ofbrain

140. River blindness may affect up to 40% ofresidents in Nigerian communities. It is due to: (choose one)

A. a tropical virus affecting the optic nerve B. onchocerciasis affecting skin and eyes C. echinococcal cyst in occipital lobe ofbrain D. tropical sprue leading to vitamin A deficiency E. oncogenic Epstein-Barr virus causing CNS lymphoma

141. The first step in dealing with a needlestick injury ofa healthcare worker from a suspected hepatitis B patient is to: (choose one)

A. administer HBIG B. determine seropositivity ofpatient and seronegativity ofhealthcare worker C. administer hepatitis vaccine D. measure SGOT levels ofpatient E. administer interferon to healthcare worker

345 QUESTIONS

142. Actinomycosis causes human disease primarily: (choose one)

A. as an allergic response to fungus B. as a disseminated bloodbome pathogen C. as an infection ofthe head, neck and chest D. as an abdominal mass E. as a cellulitis ofthe lower extremity

143. The OSHA (Occupational Safety Health Administration) rules prohibit: (choose one)

A. double gloving for mv patients B. handwashing after degloving C. recapping ofneedles D. safety goggles for potential splashes

144. Rocky Mountain spotted fever resembles measles (rubeola) clinically EXCEPT for: (choose one)

A. fever B. conjunctivitis C. rash D. Koplik spot E. age

145. The parvovirus ofFifth disease also causes: (choose one)

A. sickle cell anemia B. leukemia C. aplastic anemia D. lymphoma E. disseminated intravascular coagulation

346 QUESTIONS

146. Risk factors for lower extremity erysipelas include all but: (choose one)

A. diabetes B. lung cancer C. obesity D. venous stasis E. athlete's foot

147. One ofthe following unique advantages ofthe quinolone antibiotic is: (choose one)

A. its oral effectiveness against Pseudomonas B. its usefulness in COPD C. its usefulness in UTI D. its interaction with theophylline E. its cost

148. The acquisition ofmV is facilitated by all EXCEPT: (choose one)

A. coexistent herpes B. coexistent syphilis C. receptive anal intercourse D. condom use

149. Toxoplasmosis ofthe brain in AIDS patients is often confused with: (choose one)

A. lymphoma B. cryptococcal meningitis C. mv dementia D. vincristine neuropathy E. cerebral atrophy

347 QUESTIONS

150. A 25-year-old black male hospitalized on the psychiatric floor develops high fever and confusion. CBC shows leukocytosis. You want to consider all EXCEPT:

A. pneumococcal sepsis due to underlying B. Salmonella sepsis due to underlying my disease C. Salmonella sepsis due to underlying sickle cell disease D. neuroleptic malignant syndrome from antipsychotic medication E. clozapine-induced WBC dyscrasia

151. Infections requiring intact healthy macrophages for containment include all but one:

A. Mycobacterium tuberculosis B. brucellosis C. tularemia D. my

152. Malaria and sickle cell anemia are endemic in the same parts ofAfrica because:

A. both genomes are transmitted by the same species ofmosquito B. treatment ofmalaria is more efficacious in sickle cell patients, leading to a survival advantage C. sickled cells are resistant to malarial infection D. sickle cell patients are frequently hospitalized, protecting them from the bite ofthe mosquito

153. A 29-year-old black male presents with odynophagia (painful swallowing). You notice a 20 pound weight loss in his chart and a history ofanorectal disease. He denies homosexual activity. Physical exam reveals oral thrush. His underlying disease is: (choose one)

A. peptic acid-induced stricture B. herpes esophagitis C. candida esophagitis D. AIDS/HIY E. esophageal carcinoma

348 QUESTIONS

154. Allergic rhinitis may be distinguished from the common cold by which one ofthe following?

A. improvement with decongestants B. presence ofsneezing and coughing C. presence ofclear nasal discharge D. presence ofpale swollen nasal mucosa E. occurrence during spring and fall

155. Entamoeba histolytic may be distinguished from ulcerative colitis by the presence of:

A. acute colitis B. flask-shaped ulcers on biopsy with eosinophilic infiltration C. toxic megacolon D. chronic diarrhea E. jaundice

156. Simultaneous jaundice and renal failure are seen in all EXCEPT:

A. leptospirosis B. Rocky Mountain spotted fever C. carbon tetrachloride poisoning D. carbon monoxide poisoning E. thrombotic thrombocytopenia purpura

157. Diarrheal syndromes with resulting arthralgia/arthritis include all EXCEPT:

A. Yersinia B. intestinal bypass for obesity C. Campylobacterjejuni D. quinidine diarrhea E. inflammatory bowel disease F. Salmonella

349 QUESTIONS

158. Chagas disease causes "sleeping sickness" and cardiomyopathy. The responsible agent is a(n): (choose one)

A. neurotropic virus B. Mycobacterium C. HTLV-l found in the Caribbean D. trympanosome E. fluke

159. Schistosoma haematobium is a fluke which is least likely to cause:

A. hematuria B. bladder cancer C. decreased bladder contractility and capacity D. pulmonary hypertension E. painful micturition F. portal fibrosis

160. Physicians caring for patients returning from the 1992 Barcelona Olympics were warned of: (choose one)

A. toxic oil syndrome originating in Spain B. liver disease resulting from steroid use in atWetes C. rhabdomyolysis in runners D. penicillin-resistant pneumococci E. Legionnaires' disease from water vapors

161. Grouped vesicles should suggest which one?

A. Herpes zoster B. Herpes simplex C. Pityriasis rosea D. drug reaction

350 QUESTIONS

162. Ebola virus causes initial symptoms of:

A. myalgia, blindness B. headache, vomiting, diarrhea C. vesicular skin rash D. edema and renal failure E. edema and

163. Treatment ofpenicillin-resisitant pneumococcal meningitis can be undertaken with :

A. Methicillin B. Extended spectrum cephalosporin C. Tetracycline D. High-dose penicillin E. Erythromycin

164. Which ofthe following is the most important risk factor for community-acquired pneumonia?

A. Heart failure B. Diabetes C. COPD D. Alcoholism E. Cancer

165. The following agent has been found in clinical trials to be useful in treating the cachexia/anorexia ofAIDS patients:

A. prednisone B. marIjuana C. megestrol acetate D. hydrocortisone E. interferon

351 QUESTIONS

166. This same agent has been found to have all ofthe following side effects EXCEPT:

A. secondary adrenal insufficiency on withdrawal B. weight gain C. fluid retention D. Cushing's syndrome E. worsened glucose tolerance

167. Clarithomycin may have significant drug-drug interactions with all EXCEPT:

A. terfenadine (Seldane) B. aztemizole (Hismanal) C. theophylline D. zidovidine (AZT) E. carbamazepine

168. All ofthe following statements are true about Pneumocystis carinii pneumonia EXCEPT

A. Around 20,000 cases ofpneumocystis pneumonia develop in the US annually B. The standard treatment has been trimethoprim• sulfamethoxasole or intravenous pentamidine for 21 days C. Adjunctive prednisone therapy is continued for three months D. Adverse effects oftrimethoprim-sulfamethoxosole lead to treatment discontinuation in up to 57% ofcases E. Adverse effects ofintravenous pentamidine lead to discontinuation oftreatment in 14-55% ofcases F. Oral atovaqone has a similar success rate for mild and moderate P. carinii pneumonia with fewer treatment-limiting adverse events

352 QUESTIONS

169. Patients with CD4 counts fewer than 250/mm3 in whom dapsone was used to prevent Pneumocystis carinii also showed protection against:

A. cerebrallymphoma B. cerebral malaria C. toxoplasmic encephalitis D. mv dementia E. Mycobacterium avium complex

170. Sepsis is associated with the development of adult respiratory distress syndrome. Efforts to prevent this syndrome have been successful with the early use of:

A. intravenous dobutamine B. intravenous ketoconazole C. intravenous dopamine D. intravenous methylprednisone E. inhaled magnesium

171. Universal precautions should be used in which setting? :

A. The care ofall patients B. A doctor's order C. Only patients diagnosed with mv D. Only body fluid spill

172. Most injuries caused by needles, scalpels and other sharp instruments/devices are caused by:

A. Failure to dispose ofsharps properly and promptly B. Using hands rather than instruments to clean a tray ofsharps C. Recapping needles and not using one handed scoop method to recap D. All ofthe above

353 QUESTIONS

173. Gloves will be worn anytime you expect to touch a patient's:

A. Body fluid B. Mucous membrane C. Equipment soiled with body fluids D. All ofthe above

174. Judgments about whether or not to wear protective equipment should be based upon:

A. The cost ofthe protective items B. The diagnosis ofthe patient C. The opportunity ofcontact with blood or body fluids D. All ofthe above

175. Required infection control measures are:

A. Practice handwashing before and after each patient contact B. Teach patients to use disposable tissues for coughing and to dispose ofthem C. Remove gloves after contact with contaminated equipment and wash hands before touching the environment D. All ofthe above

176. The rights and privacy and confidentiality ofpatients regarding mY/AIDS care will be upheld by which ofthe following measures?

A. All the releases ofinformation are governed by the Privacy Act B. All releases ofinformation will be performed through the appropriate hospital channels C. Patient's consent is required for serological screening (HIV) D. All ofthe above

354 QUESTIONS

177. Ifan employee is exposed to blood or body fluids, he/she must report any needlesticks, mucus membrane/cutaneous blood exposure or potential exposure to the Employee Health physician and infection control practitioner. All ofthe following are done, but which is to be done first?

A. Wash skin surface, change clothing soiled with blood B. Go to Employee Health Clinic or Emergency Department C. Document the incident in writing D. Inform the Infection Control Practioner

178. Infection control measures required for all patients/equipment include:

A. Disinfect blood and body fluid spills with tuberculocidal cleaner B. Prevent cross-contamination ofequipment and environmental surfaces by disinfection between patients C. When in doubt contact the Infection Control Practitioner D. All ofthe above

179. What type ofmask/respirator is used by the employee to protect him/herselfwhen entering a tuberculosis isolation room?

A. Cloth mask B. HEPA (High efficiency particulate air) respirator C. Surgical mask D. None ofthe above

180. TB patients with cavitary lesions are the most infectious:

---True ---False

355 QUESTIONS

181. TB is an airborne disease and is not transmitted by contact ofthe environment such as the light switch, the walls or bedside tables:

---True ---False

182. Suspect TB patients are isolated in rooms with ventilation to the outside and with negative air flow:

---True False

183. Most TB patients become noninfectious after 1-3 weeks ofappropriate drug therapy

True False ------'

184. The causes ofupper GI bleeding include:

A. peptic ulcer disease B. esophageal varices C Mallory-Weiss tears D. all ofthe above E. none ofthe above

185 Angiodysplasia ofthe large bowel is usually asymptomatic:

---True ---False

186. The most common site ofpeptic ulcer disease is in the:

A. duodenum B. gastric fundus C. esophagus D. gastric antrum

356 QUESTIONS

187. The most probable etiology ofdiarrhea in a patient with significant bloating and abdominal pain who recently travelled to the mountains would be:

A. Giardiasis B. amebic enteritis C. traveler's diarrhea D. Staphylococcal food poisoning

188. Malabsorption could be caused by:

A. bile salt deficiency B. pancreatic insufficiency C. mucosal ~bnormalities D. lactase deficiency E. all ofthe above

189. A colonic biopsy showing transmural inflammation skip areas, noncaseating granulomas, and cobblestoning on gross appearance would most likely represent:

A. ulcerative colitis B. Crohn's disease C. celiac sprue D. tropical sprue

190. Sclerosing cholangitis and pericholangitis are most associated with:

A. ulcerative colitis B. Crohn's disease C. celiac sprue D. tropical sprue

191. The most common causes ofacute pancreatitis in the USA are biliary tract diseases and alcoholism:

---True ---False

357 QUESTIONS

192. A 40-year-old white female presenting with jaundice, pruritis and laboratory abnormalities revealing a very high alkaline phosphatase, high cholesterol, high bilirubin, and a positive anti-mitochondrial antibody would most likely have:

A. primary biliary B. Wilson's disease C. hemochromatosis D. hepatoma

193. Precipitants ofhepatic encephalopathy include:

A. sedatives B. an increased dietary protein load C. gastrointestinal bleeding D. diuretics E. all ofthe above

194. A 35-year-old white male presented to the ED with epigastric pain and vomiting. A blood specimen sent to the lab came back with "milky serum" noted on the report. His abdominal pain is likely due to:

A. peptic ulcer disease B. stomach cancer C. irritable bowel syndrome D. incarcerated inguinal hernia E. pancreatitis

195. The above patient has "milky serum" visible on fundoscopy likely due to:

A. high HDL cholesterol B. high LDL cholesterol C. high triglycerides D. low LDL cholesterol E. recent ingestion offatty meal

358 QUESTIONS

196. A common accompanying disease ofthe above situation is:

A. diabetes mellitus B. chronic calcific pancreatitis C. ulcerative colitis D. Crohn's disease E. hyperthyroidism

197. A 56-year-old white male has been anticoagulated for recurrent deep venous thrombosis. While on Warfarin he experiences a GI bleed. Which one ofthe following is false?

A. The bleeding is due to Warfarin and need not be further explored B. Anticoagulation may have unmasked a colon cancer or polyp C. The patient may have an underlying protein deficiency D. The patient may have a hypercoagulable state due to an occult carcinoma E. The patient should have a total body CT scan

198. The serum gastrin is high in all but which one ofthe following disorders?

A. perruclous anemIa B. achlorhydria C. Zollinger-Ellison syndrome D. esophageal cancer E. gastrojejunostomy with retained antrum

199. A 70-year-old woman with congestive cardiomyopathy presents with crampy left abdominal pain, weight loss, and a positive stool hemoccult. "Thumbprinting" distal to the splenic flexure is seen on X-ray. Her diagnosis is:

A. colorectal cancer B. diverticulitis C. inflammatory bowel disease D. volvulus ofsigmoid E. ischemic colitis

359 QUESTIONS

200. A 27-year-old male presents with diarrhea, right lower quadrant pain, and postprandial cramping oftwo months' duration. He has lost 10 pounds. Exam reveals a tender right lower quadrant and a positive stool hemoccult. His diagnosis is:

A. colorectal cancer B. diverticulitis c. inflammatory bowel disease D. volvulus ofsigmoid E. ischemic colitis

201. A 64-year-old white male with a history ofcolon polyps presents with dull left lower quadrant pain and pain on sitting. He notes his stools have been difficult to pass and have been blood tinged. His diagnosis is:

A. colorectal cancer B. diverticulitis C. inflammatory bowel disease D. volvulus ofsigmoid E. ischemic colitis

202. An 18-year-old girl presents with micrographia and slurred speech. A cousin had recently undergone a liver transplant. Her diagnosis is:

A. Early onset familial Parkinson's B. Toxoplasmosis C. Wilson's disease D. Hansen's disease E. Hodgkin's disease

360 QUESTIONS

203. A 35-year-old white male alcoholic had abnormal liver functions presumed secondary to drinking. An astute medical student noted that he had grayish skin and abnormal glucose metabolism. His diagnosis is:

A. alcoholic hepatitis B. Wilson's disease C. alcoholic pancreatitis D. sclerosing cholangitis E. hemochromatosis

204. All ofthe following cause pigment (calcium bilirubinate) gallstones EXCEPT:

A. hereditary spherocytosis B. hereditary ovalocytosis C. sickle cell disease D. major E. autoimmune Coombs' positive anemia

205. Risk factors for cholesterol gallstone formation include all but one ofthe following:

A. vagotomy and pyloroplasty B. female C. fecund D. middle age E. rapid weight loss F. carcinoma ofgallbladder G. lipid lowering drugs

361 QUESTIONS

206. A young white woman with a known history of Graves' disease presents with , spider angiomata, elevated liver functions, and arthralgias. She is likely to have:

A. alcoholic liver disease B. fatty liver ofpregnancy C. lupoid hepatitis D. inflammatory bowel disease E. complication ofradioiodine for Graves'

207. The characteristic laboratory abnormalities of primary biliary cirrhosis are:

A. elevated alkaline phosphatase, cholesterol, antimitochondrial antibody B. elevated alkaline phosphatase, cholesterol, smooth muscle antibody C. elevated serum carotene, protime, direct bilirubin D. elevated serum carotene, protime, indirect bilirubin E. elevated cholesterol, low albumin, elevated creatinine

208. Medical management ofportal hypertension includes the use of:

A. captopril B. aspmn C. diltiazem D. nifedipine E. propranolol

209. Patients with known esophageal varices often bleed from other than their varices. A common (50%) site is from:

A. gastritis B. duodenal ulcer C. Mallory Weiss tear D. gastric polyps E. gastric cancer

362 QUESTIONS

210. A middle-aged male has a 26-year history ofdiabetes with complications ofburning feet and retinal hemorrhage. His chiefcomplaint today is nausea and vomiting ofundigested food. After being NPO (nothing per os) for tests, ultrasound shows a full stomach. His diagnosis is:

A. Peptic ulcer disease B. Acute cholecystitis C. Gastric outlet obstruction D. Gastroesophageal reflux disease E. Autonomic neuropathy

211. Regimens used to treat this condition include all but:

A. Ranitidine B. Erythromycin C. Propulcid D. Reglan E. Multiple small meals F. Lomotil

212. A previously healthy young woman developed sudden onset ofexplosive diarrhea with crampy abdominal pain and low-grade fever. Bloody stools developed after several hours but all symptoms resolved by the following day. The most likely diagnosis is:

A. Crohn's disease B. Ulcerative colitis C. Rectal cancer D. Helicobacter pylori E. Salmonella enteritis

363 QUESTIONS

213. A 39 year old pharmaceutical representative was scheduled to leave town on vacation when he had the onset ofteft lower quadrant pain followed hours later by chills and fever. He self-medicated with his company's latest quinolone antibiotic for "prostatitis" and drove off. Hours later severe abdominal pain and nausea forced him to a local emergency department. When free air was found on abdominal film he was taken to surgery where the following was found:

A. Pyelonephritis B. Perforated duodenal ulcer C. Perforated diverticulum D. Perforated appendix E. Urethral stone F. Ischemic colon with perforation

214. All ofthe following conditions modify the presentation ofan acute abdomen EXCEPT:

A. Elderly age B. Paraplegia C. Narcotics D. Antibiotics E. Corticosteroids F. Chemotherapy G. Gender

215. A 60 year old woman has rheumatoid arthiritis requiring nonsteroidal medication and a history ofiron deficiency and GI bleeds despite H2 blocker and omeprazole therapy. She is a candidate for:

A. Misoprostol B. Vagotomy and pyloroplasty C. Propulcid D. Jejunostomy E. Combined H2 blockers and iron

364 QUESTIONS

216. Which ofthe following statements regarding Hepatitis E is false?

A. It is an enterovirus causing acute disease only B. It is spread in drinking water C. It has been seen in developed countries in Indian and Pakastani nationals returning from endemic areas D. It is a major cause ofcirrhosis worldwide E. It causes DIC and death in pregnant women

217. The mechanism ofaction ofomeprazole and lansoprazole includes all but one ofthe following:

A. It is a proton pump inhibitor B. It is an H2 receptor antagonist C. It acts by inhibiting the enzyme H+K+ ATPase D. It blocks secretion ofacid from gastric parietal cells E. It blocks basal and stimulated acid secretion F. It is the drug ofchoice for Zollinger-Ellison syndrome

218. Fears ofside effects from long-term administration ofomeprazole include:

A. Carcinoid tumors B. Gastric tumors C. Pernicious anemia D. Unknown side effects from prolonged hypochlorhydria and hypergastrinemia E. Recurrent peptic ulcer disease

219. The critical sinusoidal portal pressure below which ascites does not form is:

A. 5 mmHg B. 8mmHg C. lOmmHg D. 12mmHg E.15mmHg

365 QUESTIONS

220. The effects oftransjugular peritoneal portosystemic stent shunt include all EXCEPT:

A. Decrease sinusoid pressure B. Increase cardiac output C. Promote natriuresis D. Increase creatinine clearance E. Improve refractory ascites F. Improve hepatic encephalopathy

221. An atypical finding in gastroesophageal reflux disease is:

A. Heartburn B. Belching C. Dysphagia D. Dental erosion E. Odynophagia F. Chest pain

222. The agent shown to be best at healing ofesophagitis, decreasing need for dilatation in patients with stricture formation, and preventing relapse ofreflux esophagitis is:

A. Ranitidine B. Cimetidine C. Famotidine D. Omeprazole E. Magaldrate

223. An agent shown to be useful in treating Crohn's disease is:

A. Methotrexate B. Cyclosporine C. Inteferon D. Interleukin E. Cyclophosphamide

366 QUESTIONS

224. An agent shown to be useful in treating ulcerative colitis is:

A. Methotrexate B. Cyclosporine C. Interferon D. Interleukin E. Cyclophosphamide

225. The following drug has been associated with decreased risk for colorectal cancer:

A. Aspirin B. VitaminE C. Vitamin D D. Vitamin A E. Accutane

226. Helicobacter pylori has been associated as etiologic in all ofthe following EXCEPT:

A. Chronic antral Gastritis B. Gastric adenocarcinoma C. Nonulcer dyspepsia D. Duodenal ulcer

227. The following statements are true with regard to Helicobacter pylori EXCEPT:

A. Triple therapy with bismuth, metronidazole, and either tetracycline or amoxicillin is successful in 73-94% ofpatients B. Side effects are seen in 45% ofpatients on dual or triple therapy and are usually minor C. Relapse rates for duodenal ulcer after triple therapy at one year are 60- 80% D. Helicobacter pylori is now considered the most common cause ofduodenal ulcer E. Peptic ulcer patients who are infected with Helicobacter pylori should undergo antimicrobial treatment regardless ofwhether the is the first presentation or a recurrence

367 QUESTIONS

228. Gastric acid is necessary for the absorption ofwhich drug?

A. Ketoconazole B. Penicillin G C. Accutane D. Vitamin B 12 E. Tranexamic acid

229. Which ofthe following statements is true regarding interferon treatment ofchronic hepatitis B?

A. Short-term therapy decreases long-term viral replication B. Treatment changes the natural history ofthe disease C. Biochemical tests and liver histology were improved with high dose therapy over placebo D. Biochemical tests and liver histology were improved with low dose therapy over placebo

230. The genetic defect responsible for sickle cell anemia is:

A. Absence ofone functional beta-globin chain B. Absence ofone functional alpha-globin chain C. Substitution ofglutamic acid for valine on the beta chain D. Substitution ofvaline for glutamic acid on the beta chain E. Substitution ofglutamic acid for valine on the alpha chain F. Substitution ofvaline for glutamic acid on the alpha chain

231. Myeloproliferative disorders and myelodysplastic disorders can resemble each other. The primary difference is:

A. Myelodysplasia has a genetic component B. Myelodysplasia produces cells which are dysfunctional C. Myelodysplasia terminates in acute leukemia D. Myelodysplasia is a bone marrow disease E. Myelodysplasia responds to treatment

368 QUESTIONS

232. Myeloproliferative disorders involve a common stem cell and are named by

A. The chromosome involved B. The gene involved C. The cell line predominantly involved D. The FAB leukemia type in which it terminates E. The Rai lymphoma stage in which it terminates

233. Toxicities/undesirable side effects ofthe colony stimulating factors include all but which?

A. Bone pain B. Capillary leak syndrome C. Stimulation ofacute myeloid leukemia blasts D. Inflammatory recall syndrome E. Splenic atrophy

234. The oncogene most frequently altered in colorectal carcinoma is:

A. APC (adenomatous polyposis coli gene) B. N-myc C. bcl-2 D. c-abl E. K-RAS

235. What percentage ofpatients with Hepatitis B become chronic carriers?

A. 5% B. 10% C. 15% D.20% E.25%

369 QUESTIONS

236. What percentage of patients with Hepatitis C develop chronic disease?

A. 20% B. 40% C.60% D.80% E. 100%

237. Liver fibrosis is an known complication of:

A. Methotrexate B. Valproic acid C. Aspirin therapy ofjuvenile rhematoid arthritis D. Alphamethyldopa E. Rifampin

238. Sudden loss ofvision in one eye, as the only symptom, is compatible with:

A. B. seizure disorder c. cerebellar hemorrhage D. astrocytoma E. carotid embolization

239. The differential diagnosis ofproptosis includes all but:

A. diabetic retinopathy B. Graves' ophthalmopathy C. retrobulbar lymphoma D. orbital cellulitis E. normal variant

370 QUESTIONS

240. Periorbital edema is associated with all ofthe following EXCEPT:

A. anasarca B. superior vena cava syndrome C. Addisonian crisis D. glomerulonephritis E. anaphylaxis F. congestive heart failure

241. The earliest retinal lesion in diabetics is:

A. waxy exudates B. cotton wool spots C. flame hemorrhages D. microaneurysms E. neovascularization

242. The diabetic retinal lesion requiring laser therapy is:

A. waxy exudates B. cotton wool spots C. flame hemorrhages D. microaneurysms E. neovascularization

243. A 40-year-old male patient presents to the emergency room having had a motor vehicle accident on the interstate highway. He stated that he never saw the car in the next lane. Visual field testing reveals tunnel vision. You should consider:

A. pituitary tumor B. pituitary apoplexy C. diabetic retinopathy D. cerebrovascular accident E. optic nerve glioma

371 QUESTIONS

244. A 35-year-old black female has a history of interstitial lung disease and lymphadenopathy showing non-caseating granulomas on biopsy. She comes in now complaining ofa red eye with visual loss. Her latest problem is:

A. viral conjunctivitis B. Graves' disease C. sarcoid uveitis D. tuberculous uveitis E. histoplasmosis

245. A 25-year-old immunocompromised white female is receiving total parenteral nutrition via a central line. She is spiking daily fevers though no septic focus has been found. From your daily fundoscopic exam, you astutely make the correct diagnosis of:

A. cytomegalovirus retinopathy B. diabetic retinopathy C. cholesterol embolization D. fungal sepsis from central line E. hypersensitivity to central line plastic

246. A 37-year-old HIV positive white male comes to you because ofa painful facial rash involving one side ofhis forehead. You diagnose Herpes zoster and consider immediate ophthalmologic referral if:

A. the rash involves the cheek below the eye implying involvement ofmore than the first trigeminal division B. the rash involves the tip ofthe nose implying nasociliary ganglion involvement C. the rash involves the entire body, implying dissemination D. the rash has spread across the midline making zoster less likely E. the rash is atypically hemorrhagic implying thrombocytopenia

372 QUESTIONS

247. A 29-year-old black female presents with double vision. Exam reveals a left sixth nerve palsy. The differential diagnosis includes all but one ofthe following:

A. diabetic neuropathy B. Graves' disease C. anorexia nervosa D. pseudotumor cerebri E. Lyme disease

248. Ptosis ofthe eyelid suggests:

A. third nerve involvement B. expanding aneurysm ofinternal carotid artery C. diabetic neuropathy D. myasthenia gravis E. all ofthe above

249. A 35-year-old white female presents with complaints ofheadaches characterized by band• like pressure around the forehead and back ofneck. They occur daily and are improved with simple analgesic. The onset ofthe headaches correlated temporally with marital separation. The best diagnosis is:

A. migraine headache B. cluster headache C. chronic daily headache D. stress E. depression

250. The best technique for managing this patient's headache is:

A. counseling B. addition ofa nonsteroidal antiinflammatory C. addition ofa tricyclic D. addition ofa serotonin re-uptake inhibitor E. withdrawal ofall pain medication and institution ofa tricyclic for prophylaxis

373 QUESTIONS

251. A 42-year-old white female executive complains ofgrouped ulcerations around the lip when under periods ofstress at work. The most likely diagnosis is:

A. herpetic whitlow B. Herpes zoster C. Herpes simplex D. aphthous ulcers E. trigeminal neuralgia

252. A 57-year-old overweight hypertensive male smoker presents to the emergency room complaining ofchest pain and a sense ofimpending doom. The latter symptom raises the likelihood ofall but one ofthe following:

A. myocardial infarction B. pulmonary embolus C. dissecting aortic aneurysm D. cervical spine arthritis E. ventricular tachycardia, sustained

253. The diagnosis ofirritable bowel syndrome requires all but one ofthe following:

A. biopsy proven colonic inflammation B. alternating diarrhea and constipation C. no structural bowel disease D. no lactase deficiency E. no parasitic infiltration

254. The following patients are at increased risk ofsudden death EXCEPT:

A. a 65-year-old black male with diabetes and peripheral vascular disease B. an 83-year-old healthy white male whose wife of60 years just died C. a 50-year-old hypertensive white male who just received word that he was passed over for a much wanted promotion D. a 37-year-old female who just gave birth E. a 42-year-old schizophrenic woman whose mother and aunt both died suddenly and unexpectedly at age 42

374 QUESTIONS

255. A young "up-and-coming" politician comes to you because ofnervousness prior to speeches. This performance anxiety can be prevented with:

A. small doses ofalcohol prior to speech B. small doses ofpropranolol prior to speech C. small doses ofbenzodiazepine prior to speech D. memorizing the speech E. practicing the speech

256. A key symptom ofdepression is:

A. anorexia B. early morning awakening C. multiple aches and pains D. crying spells E. anhedonia

257. The most stressful setting is:

A. feeling fatigued from too much work B. feeling lost in a new environment C. feeling powerless to change the environment D. feeling happy in a new relationship E. feeling sad at the end ofa relationship

258. Alcoholics drink because:

A. alcohol makes them feel "normal" or good B. they are weak C. they have no self-discipline D. they have a well defined, established chemical abnormality E. they learned to do it to obtain secondary gain

375 QUESTIONS

259. The most successful treatment program for an alcoholic is:

A. Antabuse B. behavioral modification C. a 12 step Alcoholic Anonymous group D. combination therapy with clonidine and Librium E. psychoanalysis

260. The common causes ofmicrocytic hypochromic anemia are:

A. anemia B. thalassemia

C. B 12 and folate deficiency D. alcoholism E. (A) and (B) only

261. Salmonella osteomyelitis is associated with:

A. sickle cell anemia

B. B 12 and folate deficiency C. autoimmune D. iron deficiency anemia

262. Disseminated intravascular coagulation (DIC) is a result of

A. sepsis B. endothelial damage C. bum patients D. obstetrical catastrophe E. all ofthe above

376 QUESTIONS

263. Symptoms of the hyperviscosity syndrome include all ofthe following EXCEPT:

A. blurred vision B. headaches C. weakness D. dyspnea E. gangrene

264. The initial treatment ofin-hospital, severe, symptomatic hypercalcemia in a cancer patient is:

A. normal saline hydration B. furosemide C. steroids D. phosphates

265. The classic laboratory findings in iron deficiency are:

A. low MCV, low iron, low total iron- binding capacity, low B. low MCV, low iron, high total iron- binding capacity, low ferritin C. low MCV, low iron, low total iron- binding capacity, high ferritin D. high MCV, low iron, low total iron-binding capacity, high ferritin

266. Iron deficiency and thalassemia minor share all but one ofthe following:

A. microcytosis B. hypochromia C. poikilocytosis D. low serum iron E. low ferritin

377 QUESTIONS

267. Patients with sickle cell anemia are at risk for all ofthe following EXCEPT:

A. pneumococcus infections B. Salmonella infections C. hemosiderosis D. aseptic necrosis ofbone E. malaria

268. All ofthe following are features ofthrombotic thrombocytopenic purpura EXCEPT:

A. fever B. renal failure C. hemolytic anemia D. diabetes E. thrombocytopenia

269. One of the following statements about autoimmune thrombocytopenic purpura is false:

A. The spleen is markedly enlarged B. The bone marrow contains many megakaryocytes C. The spleen is hot on nuclear scan D. Petechiae are seen in dependent locations and mucous membranes E. Platelet transfusion rarely results in significant rise in platelet count

270. A young man presents with a second episode ofspontaneous deep vein thrombosis. Before heparin is begun, all ofthe following tests are indicated EXCEPT:

A. protein S level B. protein C level C. CT scan ofabdomen looking for cancer D. antithrombin III level E. partial thromboplastin time

378 QUESTIONS

271. Thrombocytosis is associated with all ofthe following EXCEPT:

A. chronic myelogenous leukemia B. acute myelogenous leukemia C. polycythemia rubra vera D. iron deficiency E. rheumatoid arthritis F. essential thrombocythemia

272. A 69-year-old black male has had a myelodysplastic syndrome for two years requiring blood transfusions and antibiotics for frequent infections due to neutropenia. He presents now with fever, increasing numbers ofblasts (50%), skin petechiae, necrotic material in his nares, and space occupying lesions ofliver on CT scan. His clinical picture is most compatible with:

A. aplastic anemia B. paroxysmal nocturnal hemoglobinuria C. myelofibrosis D. acute leukemia with disseminated aspergillosis E. metastatic carcinoma ofunknown primary

273. Patients with sickle cell anemia chronically hemolize and are moderately anemic. A precipitous drop in hemoglobin in the absence ofovert blood loss should lead you to consider which treatment?

A. Plasma infusion B. Pyridoxine C. Folate D. Iron E. Vitamin B12

379 QUESTIONS

274. Which ofthe following heterozygous states has the potential for severe disease, except in blacks when it is usually mild?

A. minor B. sickle-C C. sickle trait D. sickle-beta thalassemia E. alpha thalassemia

275. Which ofthe sickle syndromes is known for its ocular complications?

A. sickle-C B. sickle trait C. sickle-beta thalassemia D. sickle cell anemia

276. Which ofthe following myeloproliferative disorders is most likely to have associated gout?

A. Myelofibrosis B. Chronic myelogenous leukemia C. Essential thrombocytosis D. Polycythemia rubra vera

277. Disseminated intravascular coagulation with consumption coagulopathy is seen in all but which setting?

A. Complications oflabor and delivery B. Thrombotic thrombocytopenic purpura C. Bums D. Sepsis E. With Denver shunts in ascites management

380 QUESTIONS

278. Match the clinical feature on the left with the disease on the right:

A. symptoms ofhyperviscosity 1. Sickle-C B. WBC of300,000 2. myelofibrosis C. reticulocyte count of60% 3. sickle trait D. Hct of 15%, chipmunk facies 4. thalassemia major E. platelet count of 1.5 million 5. glucose-6-phosphate dehydrogenase deficiency F. lymphocyte count of 100,000 6. polycythemia rubra vera G. target cells, Hb crystals 7. chronic lymphocytic leukemia H. balanced polymorphism 8. Hemoglobin C I. pregnancy complications 9. chronic myelogenous leukemia 1. teardrop cells 10. essential thrombocytosis

279. Effects ofcolony stimulating factors (G-CSF and GM-CSF) include all but which?

A. Bone pain B. Stimulation ofleukemic blasts C. Stimulation ofneutrophil production D. Stimulation ofred cell production E. Recall ofinflammatory pericarditis

280. The basic defect in thalassemia major (Cooley's anemia) is:

A. Deletion ofone alpha-globin gene B. Deletion oftwo alpha-globin genes C. Defective beta-globin production by one gene D. Defective beta-globin production by two genes E. Deletion ofone alpha-globin gene and defective beta-globin production by one gene

281. Hereditary spherocytosis is due to a defect in:

A. Hemoglobin, causing crystals ofhemoglobin to form B. Membrane production C. Splenic destruction ofred cells D. Hepatic metabolism leading to calcium bilirubinate gallstones E. Folate deficiency

381 QUESTIONS

282. The unique features ofhairy cell leukemia are:

A. Hairy cells which are positive for tartrate-resistant acid phosphatase B. Hairy cells which are negative for tartrate-resistant acid phosphatase C. Hairy cells which are positive for tartrate-sensitive acid phosphatase D. Hairy cells which are negative for tartrate-sensitive acid phosphatase

283. American Burkitt lymphoma, undifferentiated, B cell type preferentially affects which organs?

A. Head and neck B. Spleen and liver C. Lymph nodes and liver D. Brain and thyroid E. Pelvis and ovaries

284. A young black male with a large mediastinal mass, very high circulating WBC count with blasts, and convoluted cells on biopsy ofthe mass has which kind oflymphoma?

A. B cell B. Pre-B cell C. T cell D. Undifferentiated E. HTLV-I related

285. What percentage ofpatients with multiple myeloma have a detectable abnormal paraprotein?

A. 25% B.I% C.50% D.75% E.99%

382 QUESTIONS

286. Which type ofacute myelogenous leukemia is associated with DIC?

A. MI B.M2 C.M3 D.M4 E.MS F.M6 G.M7

287. What structure is found in the leukemic myeloid blast cell which helps to diagnose acute leukemia?

A. Microtubule B. Auerrod C. Dawn ofneutrophilia D. Nucleolus E. Owl's eye nucleus

288. All ofthe following are causes ofsecondary polycythemia EXCEPT:

A. Smoking B. Carbon monoxide C. Polycythemia rubra vera D. Congenital heart disease E. Chronic hypoxemia F. High affinity hemoglobin

383 QUESTIONS

289. Causes ofreactive thrombocytosis include all but which?

A. Iron deficiency anemia B. Inflammatory bowel disease C. Sickle cell anemia D. Rheumatoid arthritis E. Chronic myelogenous leukemia

290. All but one ofthe following are B cell tumors. Which one is not?

A. Mycosis fungoides B. Lymphocytic lymphoma C. Multiple myeloma D. Chronic lymphocytic leukemia E. Solitary plasmacytoma

291. The genetic defect in chronic granulocytic (myelogenous) leukemia is

A. Trisomy 8 B. t(8;21) C. t(15;17) D. t(9;22) E. t(1l;14)

292. Match the disease state on the left with the abnormal protein/description on the right.

A. Waldenstrom's 1. IgG > 3 grams B. Multiple myeloma 2. IgM elevated C. Benign monoclonal gammopathy 3. Incomplete gamma globulin with diarrhea D. Alpha heavy chain disease 4. IgG < 3 grams E. Solitary plasmacytoma 5. Nasopharyngeal or sinus mass

384 QUESTIONS

293. With special techniques and stains, what percentage ofacute leukemias will have chromosome abnormalities?

A. 20% B. 40% C. 55% D.75% E. 100%

294. Bone marrow transplantation has been used in a curative fashion in all EXCEPT:

A. Sickle cell anemia B. Thalassemia major C. Thalassemia minor D. Chronic phase ofchronic myelogenous leukemia E. Myelodysplasia

295. Treatment ofFactor VIII and Factor IX deficiency requires replacement ofclotting factors, which have in the past and present had side effects which include all but: (one best answer)

A. Hepatitis B. Pancreatitis C. Disseminated intravascular coagulation D. Chronic liver disease E.AIDS

296. Factor VIII and Factor IX deficiency are clinical similarly except for: (one best answer)

A. Sex-linked transmission B. Penetration C. Incidence D. Site ofbleeding E. Use ofreplacement clotting factors

385 QUESTIONS

297. Von Willebrand's disease has the following features EXCEPT:

A. Mucosal bleeding B. Abnormal platelet function C. Abnormal partial thromboplastin time D. Abnormal factor VIII function E. Bleeding into joints

298. Aspirin causes platelet dysfunction in which way (single best answer)?

A. Irreversible inhibition ofcyclo-oxygenase for the life ofthe platelet B. Irreversible inhibition ofcyclo-oxygenase for the half-life ofthe drug C. Reversible inhibition ofthromboxane A2 D. By blocking the final Vitamin K dependent step ofhepatic synthesis ofclotting factors E. By altering receptor sites on endothelial cell membranes, inhibiting the initial platelet aggregation

299. A 68 year old male presents to the clinic with low back pain unresponsive to nonsteroidals, painful tender ribs, and pallor on examination. He is found to have an elevated sedimentation rate, moderately severe anemia which is normochromic and normocytic, and rouleaux formation reported on peripheral smear. The rouleaux formation in this patient is:

A. A consequence ofthe anemia B. A result ofhigh circulating gamma globulins C. A result of polycythemia D. A manifestation ofAIDS-related lymphoma E. A result of coexisting hypercalcemia

386 QUESTIONS

300. In the above patient, appropriate studies are done and the patient is treated for pain while awaiting test results. He is brought in to the clinic urgently by his family the next day with lethargy, dehydration, constipation, and weakness. You recommend hospitalization for treatment of:

A. Hyponatremia B. Hyperviscosity syndrome C. AIDS related dementia D. AIDS related cryptococcal meningitis E. Hypercalcemia

301. An elderly white female presents to your clinic complaining ofdifficulty buttoning h~r clothes. On examination her conjunctivae are pale, her sclerae are questionably icteric, her tongue is beefy red and smooth, and she has no vibratory or proprioception sensation in her extremities. Her family also complains that she is having personality changes and they fear she may be developing Alzheimer's dementia. The diagnosis is:

A. Neuropathy due to pancreatic cancer B. Multifocalleukoencephalopathy C. Pernicious anemia D. Peripheral neuropathy due to occult alcoholism E. Folate deficiency F. Myxedema madness

302. A 29 year old male ofMediterranean descent presents to you with complaints ofa recent painful spider bite. He is developing signs oferythema and induration and you consider using dapsone for a brown recluse (fiddleback) bite. You counsel him regarding the planned treatment, institute it, and plan to bring him back soon to check: (single best answer)

A. WBC for secondary infection B. Hemoglobin for hemolysis C. The wound for need for skin grafting D. mVtest E. G6PD level

387 QUESTIONS

303. All ofthe following can cause neutropenia EXCEPT:

A. Lithium B. Propylthiouracil C. Ticlid D. Captopril E. Thorazine

304. A 65-year-old black male veteran has a history ofheavy alcohol and cigarette abuse. He is at risk for all BUT which ofthe following?

A. bladder cancer B. neurofibrosarcoma C. laryngeal cancer D. lung cancer E. pharyngeal cancer F. esophageal cancer

305. The work-up ofa patient with a prostate nodule includes all but which?

A. PSA B. prostatic ultrasound C. chemistries D. bone scan E. CT brain scan

306. A classic example ofco-carcinogenicity is:

A. dietary fat and familial polyposis B. chest wall irradiation and nulliparity C. asbestos and cigarette smoke D. sunlight and fair complexion E. papilloma virus and early sexual intercourse

388 QUESTIONS

307. A 62-year-old white male was diagnosed with gastric lymphoma, diffuse large-cell type, and chemotherapy was begun. Two weeks post treatment he developed sudden severe abdominal pain, a rigid abdomen, and absent bowel sounds. Upright film showed free air. What happened?

A. typhlitis from neutropenia from chemotherapy B. vincristine related autonomic neuropathy ofbowel C. perforation of stomach from rapidly shrinking tumor D. stress-related gastritis E. gallstone ileus

308. Which of the following cancers is least likely to have associated hypercalcemia?

A. breast cancer B. myeloma C. large-cell lung cancer D. small-cell lung cancer E. renal cell

309. Patient presents with dyspnea and facial, neck, and hand swelling. A right upper lobe mass is present on chest film. The most likely explanation for symptoms is:

A. anerma B. cancer-related glomerulonephritis C. heart failure D. superior vena cava syndrome E. inferior vena cava obstruction

310. A patient was treated for small-call lung cancer with chemotherapy ten days ago. He presents now with fever to 103 degrees, malaise, and pallor. His Hgb is 7.9 g, his WBC is 0.3, and platelets are 60,000. The most important aspect oftreatment is:

A. transfusion ofred cells B. transfusion ofwhite cells C. transfusion ofplatelets D. initiation ofintravenous fluids

389 QUESTIONS

E. initiation ofantibiotics

311. A 70-year-old black male was treated with radiation therapy for prostate cancer. After 5 weeks, he began to complain of dysuria. Urine was positive for hemoglobin and culture was negative. He likely had:

A. hemorrhagic cystitis from cyclophosphamide B. pyelonephritis C. ureteral stone D. recurrence ofprostate cancer E. radiation cystitis

312. A 25-year-old otherwise healthy white female complained ofa two-month history of right cervical mass. A pre-biopsy chest X-ray revealed a mediastinal mass. Biopsy revealed large cells with double mirror image nuclei and prominent nucleoli. A likely diagnosis is:

A. nodular sclerosing Hodgkin's disease B. squamous cell carcinoma ofthe base ofthe tongue C. cat scratch disease D. bronchial cleft cyst E. T cell lymphoma

313. A 53-year-old alcoholic smoker presented with a large right cervical mass and pain in his throat. A likely diagnosis is:

A. nodular sclerosing Hodgkin's disease B. squamous cell carcinoma ofthe base ofthe tongue C. cat scratch disease D. bronchial cleft cyst E. T cell lymphoma

390 QUESTIONS

314. A 17-year-old black male basketball player presented with fevers, a right cervical mass, and a chest film showing a mediastinal mass. His WBC was 84,000 with 60% blasts. A likely diagnosis is:

A. nodular sclerosing Hodgkin's disease B. squamous cell carcinoma ofthe base ofthe tongue C. cat scratch disease D. bronchial cleft cyst E. T cell lymphoma

315. Screening mammography in women younger than 50 is difficult because ofall EXCEPT:

A. Young women have dense breasts which may obscure malignant lesions B. Young women do not develop breast cancer at rate sufficient to warrant screemng C. Fibrocystic breast changes may present as lumps which may not be seen on mammography D. Young women tend to have aggressive tumors, the outcome ofwhich may not be changed by screening

316. Which cell type tends to produce endobronchial tumor growth and presents with hemoptysis?

A. Small cell B. Alveolar cell carcinoma C. Adenocarcinoma D. Squamous cell carcinoma E. Large cell carcinoma

391 QUESTIONS

317. The following are features ofectopic production ofACTH by small cell lung cancer EXCEPT:

A. Hypokalemia B. Buffalo hump C. Weakness D. Metabolic alkalosis E. Weight loss

318. The following are true regarding the treatment ofpremenopausal breast cancer EXCEPT:

A. Modified radical mastectomy and lumpectomy with ipsilateral breast tissue irradiation have comparable results B. Lymph node dissection is performed with curative intent C. Lymph node dissection or sampling is done to assess status ofnodes as an indicator for the need for systemic therapy ifpositive D. Tamoxifen is a useful hormonal therapy for systemic spread E. Combination chemotherapy is useful for systemic spread

319. The following drugs cause myelosuppression at usual doses EXCEPT:

A. Bleomycin B. Adriamycin C. Cyclophosphamide D. Methotrexate E. 5-fluorouracil

320. The type ofleukemia associated with DIC, particularly with treatment, is:

A. Chronic granulocytic leukemia B. Acute lymphoblastic leukemia C. Chronic lymphocytic leukemia D. Acute promyelocytic leukemia E. Blast crisis ofchronic myelogenous leukemia

392 QUESTIONS

321. The hypercoagulable state associated with several malignancies is in part due activation of:

A. Factor V B. Factor VII C. Factor VIII D. Factor IX E. Factor X

322. Amyloid deposition is seen with all ofthe following EXCEPT:

A. Multiple myeloma B. Waldenstrom's macroglobulinemia C. Medullary carcinoma ofthyroid D. Tuberculosis E. Lung cancer

323. All ofthe following are true about cigarette smoking EXCEPT:

A. It is responsible for 85% oflung cancer B. It is responsible for 30% ofall cancers C. It is respsonsible for 90% ofmalignant mesotheliomas D. It is synergistic with asbestos to cause lung cancer E. It increases the relative risk oflung cancer in non-smokers by passive exposure

324. The causes ofincreased anion gap metabolic acidosis include ALL EXCEPT:

A. methanol ingestion B. ethylene glycol ingestion C. diabetic ketoacidosis D. lactic acidosis E. paraldehyde .F. renal tubular acidosis

393 QUESTIONS

325. The most common cause ofsyndrome ofinappropriate ADH from a tumor source is:

A. small-cell carcinoma ofthe lung B. renal carcinoma C. colon cancer D. thyroid carcinoma E. breast cancer

326. Common findings with hypothyroidism include all EXCEPT:

A. hyperlipidemia B. increased CPK levels C. lethargy D. hypermenorrhea E. velvet skin

327. Primary hyperparathyroidism classically has which ofthe following lab abnormalities?

A. hypercalcemia and hypophosphatemia B. hypercalcemia and C. and hyperphosphatemia D. hypocalcemia and hypophosphatemia

328. Which endocrine disease would you most suspect in a patient with hypertension, palpitations, arrhythmias, headaches, pallor, flushing, sweating, and anxiety?

A. Pheochromocytoma B. Conn's adrenal adenoma C. Cushing's syndrome D. Addison's disease E. Ovarian failure

394 QUESTIONS

329. The leading precipitant ofdiabetic ketoacidosis is:

A. Stress B. Peptic ulcer disease C. Cholelithiasis D. Silent myocardial infarction E. Infection .

330. Initial treatment of the comatose patient should include:

A. Maintaining patent airway B. Bolus ofconcentrated glucose solution (D50) C. Naloxone therapy D. All ofthe above

331. Diabetic ketoacidosis results in increased ketone formation by all ofthe following EXCEPT:

A. Insulin lack B. Decreased lipolysis C. Increased plasma free fatty acids D. Increased liver fatty acids E. Glucagon excess

332. The most urgent aspect oftreatment ofdiabetic ketoacidosis is to rapidly:

A. Correct the acidosis B. Correct the hyperkalemia C. Infuse normal saline D. Inject large doses of regular insulin E. Correct the hypophosphatemia

395 QUESTIONS

333. Which complication ofdiabetes initially worsens with institution of "tight control"?

A. Nephropathy B. Peripheral neuropathy C. Autonomic neuropathy D. Retinopathy E. Diabetic foot ulcers

334. Diabetic renal insufficiency is heralded by:

A. Rising creatinine B. Hypertension C. Microalbuminuria D. Hematuria E. Pyuria

335. The concordance rate for inheritance oftype II diabetes mellitus in identical twins is:

A. 25% B. 45% C.33% D.lOO% E.75%

336. Symptoms and signs ofhypoglycemia include all but:

A. Facial flush B. Diaphoresis C. Anxiety D. Confusion E. Focal neurologic signs

396 QUESTIONS

337. The central consistent feature ofthe sick euthyroid syndrome is:

A. Decreased production ofT3 due to inhibition ofthe peripheral 5' monodeiodination ofT4 B. Response ofTSH to TRH stimulation C. Critically ill patients who are hypothyroid D. Mildly ill patients who are euthyroid

338. The metabolic state in simple goiter is:

A. Hypermetabolic B. Hypometabolic C. Normal

339. Hypothyroidism is a primary disease ofthe thyroid gland in what percentage ofcases?

A. 10% B. 50% C. 75% D.95% E. 100%

340. Features ofapathetic hyperthyroidism include all but:

A. Myopathy B. Cardiac failure C. Elderly D. Atrial arrhythmias E. Marked hypermetabolism

397 QUESTIONS

341. Features offlorid thyrotoxicosis include all but:

A. Weight loss with good appetite B. Tachycardia C. Loss oflateral eyebrows D. Hyperdefecation E. Heat intolerance

342. Metformin is not to be used in the treatment ofdiabetics with which:

A. Obesity B. Renal failure C. Type II diabetes D. Peripheral neuropathy E. Prior acute myocardial infarction

343. The TRH stimulation test shows what result in toxic multinodular goiter?

A. Increased TRH B. Increased TSH C. Increased TRH and TSH D. Decreased or absent TRH E. Decreased or absent TSH

344. The reason to pursue the workup ofa thyroid nodule is to:

A. Prevent hypothyroidism B. Prevent hyperthyroidism C. Diagnose malignancy D. Prevent bleeding into the nodule E. Prevent symptoms ofgoiter

398 QUESTIONS

345. Which commonly coexisting endocrine problem can complicate the management of hypothyroidism?

A. Pheochromocytoma B. Adrenal insufficiency C. Diabetes mellitus D. Diabetes insipidus E. Islet cell tumor

346. Which ofthe following thyroid cancers is familial?

A. Papillary B. Follicular C. Anaplastic D. Lymphoma E. Medullary

347. Which ofthe following is not usually associated with hyperparathyroidism due to an adenoma?

A. Neuropsychiatric symptoms B. Marked weight loss C. Increased calcium and increased PTH D. Peptic ulcer disease E. Ureteral stones

348. Which ofthe following is not associated with multiple endocrine neoplasia Type IIa?

A. Parathyroid hyperplasia B. Pheochromocytoma C. Medullary carcinoma ofthyroid D. Mucosal neuromas E. Increased calcitonin and ACTH

399 QUESTIONS

349. An elderly patient was found unresponsive by his neighbor who was a paramedic and immediately began CPR. He was brought to the ER ofyour hospital where advanced cardiac life support was instituted and a rhythm was established. He was intubated and placed in the intensive care unit. He has no wife, no children, and a physician nephew who wants an EEG and life support removed. Your moral and ethical obligation at this time (immediately-post code) to this patient is to:

A. Follow the wishes ofthe nephew and remove life support because he is a physician and blood relative B. Continue supportive care and assess the patient's clinical condition C. Advise the nephew that you are the attending physician and will make the decisions D. Call the hospital ethics committee E. Order the EEG to satisfy the physician nephew

350. The patient does not regain consciousness post-resuscitation. It is learned that he had written an advanced directive and wished no resuscitation for any condition. Your resposibility at this time is to:

A. Follow the wishes ofthe patient and remove life support B. Order an EEG to look for brain death prior to removal oflife support C. Berate the neighbor for instituting CPR in a patient who did not want CPR D. Ask for a neurology consult to assess prognosis E. Call the hospital ethics committee

351. You are asked to see a patient with advanced Hodgkin's disease on routine morphine who has failed several chemotherapy and radiation protocols. He is interested in trying yet another drug regimen but his wife wants to take him home, build him up with vitamins, and take him to the Native American medicine man. She is hostile and believes your care to be substandard. You should:

A. Honor the wife's wishes for discharge as she is next- of- kin B. Honor the patient's wishes and proceed immediately with chemotherapy C. Request a second opinion D. Discuss the proposed fourth line therapy with the patient, including realistic prospects ofundesirable side effects and low likelihood ofremission E. Notify your malpractice attorney ofan imminent lawsuit

400 QUESTIONS

352. You are caring for a patient in the intensive care unit who is having a GI bleed and requires blood. His son is a Jehovah's witness and protests, saying that you will violate his church's wishes by giving his father blood. The patient is unable to sign the consent form for blood You should:

A. Give the blood without patient consent and against the wishes ofthe son B. Contact other family members to determine patient's wishes and ifthere is a power of attorney for health care C. Withhold the blood, support with intravenous colloid and crystalloid instead D. Withhold the blood and administer morphine for palliative care E. Contact the leading pharmaceutical company testing artifical hemoglobin

353. You successfully locate the wife ofthe above patient. She informs you that the patient has agreed in the past to and she has no reason to believe he has changed his mind. You inform the son that you intend transfuse the patient, and he becomes threatening to you and the nurses. You should:

A. Yield to the wishes ofthe son B. Transfuse the patient and hope for the best C. Transfuse the patient, bar the son from the unit, and notify security D. Transfuse the patient, bar the son from the unit, notify security, and ask the chaplain to visit with the family E. Poll other family members for other opinions

354. A middle-aged male hypertensive smoker presents to the ER with chest pain which is substernal and radiates to the left arm and jaw. He is diaphoretic on exam and his EKG shows an acute myocardial injury pattern. You are writing thrombolytic orders when the nurse summons you to help stop the patient from leaving. You explain that he is having a heart attack and needs to be hospitalized for therapy, and that it is unsafe for him to leave. He insists on leaving. You should:

A. Let him go B. Call security to keep him in the hospital C. Discuss the implications ofhis leaving with the patient and family members and document in the chart D. Sedate him with an IV dose ofmorphine and transport to the unit E. Use leather restraints to prevent him from leaving

401 QUESTIONS

355. A cancer patient comes to you, wanting you to assist in his suicide. He is in constant pain and wishes to end his life. You should:

A. Prescribe barbiturates and discuss lethal doses B. Prescribe morphine for a month's therapy C. Advise him to read Final Exit D. Advise him to see his minister E. Discuss his pain, its causes, and offer a pain management plan to include hospice care

356. An elderly woman collapsed at home and after stabilization at the hospital, was found to be comatose due to a massive brainstem stroke. After one month oftherapy in the intensive care unit with no signs ofimprovement, her husband and children approach the attending physician and ask that life support be removed, citing that the patient stated many times to them that she did not want to be kept alive if persistently unconscious. The attending physician values human life, regardless ofothers' perception ofits quality, and does not want to remove the ventilator. You are involved as a consultant and the family turns to you for advice. You should:

A. Ask the attending to be reasonable and follow the family's wishes B. Ask the family to be reasonable and follow the attending's wishes C. Order an EEG D. Request a meeting ofthe hospital Ethics Committee and invite the attending to present the case for discussion E. Recommend that the family fire the attending and offer to find another to assume care ofthe patient

402 QUESTIONS

357. A young man presents to your clinic complaining ofshortness ofbreath, fever, chills, and weight loss. You notice that he is accompanied by another young man who is very attentive and you confirm that he is gay and his symptoms are likely due to Pneumocystis pneumonia. You personally feel that homosexuality is wrong and you are nervous about caring for an mv patient. You should:

A. Refer him to an infectious disease specialist with a wait period of3 days B. Inform him ofyour negative feelings about his lifestyle and your inability to care for him C. Initiate care for his pneumonia and discuss your feelings with a counselor or friend D. Review the latest literature on transmission ofmv to health care workers E. Explore your homophobia with a psychoanalyst

358. A 59 year old male with no previous medical history presents with classic pneumococcal pneumonia. He is ill with a p02 of40 but is lucid. As you begin treatment ofhis pneumonia and attempt to increase his oxygenation, he informs the nurses that he does not want to be on a ventilator or resuscitated. You explain to the patient that he has a treatable condition which might require only temporary ventilatory support. He is emphatic that he will not go on the ventilator nor be resuscitated. You should:

A. Call for a pulmonary consult B. Call for an infectious disease consult C. Call the next-of-kin to talk him into being ventilated D. Call for a psychiatric consult to establish competency E. Proceed with your antibiotic and oxygen therapy and write a NO CODE BLUE order and note

403 ANSWERS

1. D This is the presentation ofacute subarachnoid hemorrhage. Varying levels of coma may be present with prognosis declining as grade increases. (Page 5)

2. E The exam ofthe cardiovascular system reveals findings ofcoarctation. The murmur posteriorly is from increased blood flow through intercostal vessels. is from increased blood flow through intercostal vessels. The latter leads to rib notching on chest X-ray. (page 2)

3. A Most subarachnoid hemorrhages occur from aneurysms in this location. Turner's syndrome patients have associated multiple cerebral aneursyms and coarctation ofthe aorta. (page 5)

4. C (Page 2)

5. B An echocardiogram quickly confirms the clinical diagnosis of aortic stenosis and gives an estimate ofthe severity. With a valve orifice area of< 0.5 cm2 and the clinical symptoms ofsyncope secondary to heart block (Stokes-Adams), valve replacement is indicated. (page 15)

6. E When symptoms ofangina, syncope or CHF develop in a patient with aortic stenosis, the life expectancy is 50% at two years. (page 15)

7. D Cough and upper respiratory congestion in the spring can often be due to allergic rhinitis and asthma. However, the risk factors for and physical findings ofheart disease demand that it be considered. The patient had on his own sought an allergy consultation and had negative skin tests. (pages 6 -7) ANSWERS

8. D The patient's nocturnal symptoms and obesity speak for sleep apnea. However they also were witnessed in the daytime while sitting (not recumbent). Most sleep apnea patients complain ofdaytime somnolence not nighttime events. The patient could have COPD with his long smoking history. The cough was non-productive; the X-ray did not show increased A-P diameter or flattened diaphragms ofCOPD. The patient's girth and recumbent symptoms could suggest gastroesophageal reflux disease. Cough can be prominent in this symptom complex. However, LV dysfunction (with 83 gallop) in this patient caused his paroxysmal nocturnal dyspnea and his angina-equivalent during the stressful conversation about his health. He improved with diuretics and calcium channel blockers. (pages 6-7)

9. D This injection drug user has acute endocarditis as manifested by signs ofembolization, fever, and murmur. All answers are appropriate for initial work-up ofthis condition except for liver biopsy. He may well have liver disease as a sequel to hepatitis B or C from injection drug use, but the liver would not be biopsied on day one. (pages 18-19)

10. D The dreaded complication ofperforated aortic cusps occurred with the patient developing acute aortic insufficiency. Urgent management includes consideration ofvalve replacement. (page 16)

11. B Pulmonary emboli are very common after cholecystectomy. Pre-op low dose subcutaneous heparin can decrease the incidence. (pages 21,34-38)

12. E Coumadin skin necrosis occurs in predisposed individuals in areas ofadipose tissue. It occurs in protein C deficient patients when Coumadin is given before adequate heparin therapy has raised levels ofanti-thrombin III. (page 35)

13. B (page 35)

14. A (This page)

15. E (page 23)

16. B (This page)

17. C (pages 23-24)

18. A (page 17)

406 ANSWERS

19. True (page 22)

20. E (pages 22-23)

21. E (pages 1,2)

22. A (page 4)

23. B (page 4)

24. True (This page)

25. B (page 17)

26. D (page 24)

27. True (This page)

28. C (page 9)

29. B (This page)

30. A (page 11)

31. D (page 11)

32. E (page 23)

33. C (pages 23-24)

34. D (page 14)

35. C (page 14)

36. C (page 14)

37. B (page 8)

407 ANSWERS

38. A (page 9 and this page)

39. A (page 28)

40. D (page 30)

41. E (pages 60-61)

42. D (page 27)

43. D (page 33)

44. B (page 35)

45. D(Page31)

46. E (page 112)

47. F (page 31)

48. True (page 32)

49. D(Page31)

50. C (page 35)

51. B (page 41)

52. D (page 43)

53. D (page 30)

54. C (page 33)

55. C (page 27)

56. D (page 29)

408 ANSWERS

57. True (page 28)

58. E (page 29)

59. D (pages 116-117)

60. D (Page 109)

61. E (page 116-117)

62. B (pages 78, 79, 94)

63. C (pages 78, 79)

64. E (pages 78, 79)

65. C (Page 68)

66. D (page 68)

67. B (This page)

68. C (This page)

69. E (pages 74, 86)

70. C (pages 74, 86)

71. A (page 78)

72. A (Page 65)

73. E (This page)

74. B (page 74, this page)

75. C (Page 71)

409 ANSWERS

76. A (page 72)

77. D (page 65)

78. D (This page)

79. E (page 78)

80. B (pages 63, 78, 79)

81. A (page 63)

82. E (page 64)

83. D (This page)

84. E (Page 73)

85. D (pages 74-75)

86. A (Page 74)

87. B (page 74)

88. E (page 74)

89. D (page 75)

90. E (page 76)

91. B (page 77)

92. D (page 77)

93. E (page 77)

94. B (page 78)

410 ANSWERS

95. E (This page)

96. A (page 80)

97. H (This page)

98. C (page 72)

99. A (pages 132-133)

100. A (page 82)

101. D (page 93)

102. E (page 84)

103. A (page 87)

104. C (page 87)

105. E (page 88)

106. C (page 85)

107. False (page 83)

108. D (page 86)

109. B (page 90)

110. C (This page)

Ill. C (page 87)

112. D (page 63)

113. F (page 147)

411 ANSWERS

114. C (Page 232)

115. A (This page)

116. E (page 89)

117. C (page 83)

118. D (page 93)

119. B (page 87)

120. D (This page)

121. D (Pages 92, 93)

122. C (Page 86, this page)

123. A- 8 B-6 C-2 D - 3 E-4 F-l G- 5 H-7 (Pages 92, 89)

124. A (Page 109)

125. A (This page)

126. True (Page 109)

127. A2, B3, C5, Dl, E4 (Pages 120-122, 139)

128. E (Page 52)

412 ANSWERS

129. C (page 52)

130. B (pages 100-103)

131. B (page 139)

132. A (pages 53, 54)

133. B (Page 129)

134. D (page 70)

135. A (page 123)

136. A (page 124)

137. D (page 105)

138. D (pages 132, 133)

139. F (pages 97, 139)

140. B (Page 135)

141. B (Page 163)

142. C (Page 124)

143. C (This page)

144. D (page 132)

145. C (page 200)

146. B (Page 219)

147. A (This page)

413 ANSWERS

148. D. (page 103)

149. A (page 99)

150. E (pages 199,212)

151. D (page 52, this page)

152. C (Page 200)

153. D (pages 97-100)

154. D (This page)

155. B (page 127)

156. D (page 227)

157. D (pages 151, 152)

158. D (Page 135)

159. F (Page 124)

160. D (Page 111)

161. B (page 106)

162. B (page 107)

163. B (page 111)

164. C (page 116)

165. C (This page)

166. D (This page)

414 ANSWERS

167. D (This page)

168. C (page 101, this page)

169. C (page 101)

170. B (This page)

171. A (This page)

172. D (This page)

173. D (This page)

174. C (page 103)

175. D (This page)

176. D (page 297)

177. A (This page)

178. D (This page)

179. B (This page)

180. True (This page)

181. True (This page)

182. True (This page)

183. True (This page)

184. D (pages 144-147)

185. True (Page 153)

415 ANSWERS

186. A (Page 146)

187. A (Page 152)

188. E (Page 150)

189. B (Page 156)

190. A (Page 156)

191. True (Page 159)

192. A (Page 168)

193. E (Page 170)

194. E (Page 159)

195. C(Page 179)

196. A (Page 179)

197. A (Pages 38, 155)

198. D (Page 149)

199. E (Pages 157, 158)

200. C (Pages 155,156

201. A (Page 155)

202. C (Pages 166, 167)

203. E (Pages 24, 25, 166, 167

204. E (Page 168)

205. F (Page 158)

206. C (Page 166)

416 ANSWERS

207. A (Page 168)

208. E (Page 171)

209. B (Page 171)

210. E (Page 149)

211. F (Page 149)

212. E (Page 126)

213. C (Page 153)

214. G (This page)

215. A (Page 147, 148)

216. D (Page 164)

217. B (Pages 147, 148)

218. E (Page 148)

219. B (This page)

220. F (Page 171)

221. D (Page 144)

222. D (Page 148)

223. A (Page 156)

224. B (Page 156)

225. A (Page 155)

226. C (Page 146)

227. C (This page)

417 ANSWERS

228. A (This page)

229. C (This page)

230. D (Page 198)

231. B (Page 206)

232. C (Page 213)

233. E (Page 231)

234. E (Page 154)

235. B (Page 162)

236. D (Page 164)

237. A (Page 165)

238. E (Page 175)

239. A (Pages 178, 181)

240. C (This page)

241. D (Page 178)

242. E (Page 179)

243. A (page 175)

244. C (page 177)

245. D (Page 179)

246. B (page 176)

247. C (Page 180)

248. E (This page)

418 ANSWERS

249. C (Page 183)

250. E (Page 183)

251. C (Page 143)

252. D (Page 187)

253. A (Page 189)

254. D (Pages 192, 187)

255. B (Page 187)

256. E (page 191)

257. C (Page 187)

258. A (Page 188)

259. C (Page 189)

260. E (Page 193)

261. A (Page 198)

262. E (Page 227)

263. E (Page 223)

264. A (Page 224)

265. B (Page 193)

266. E (Page 193)

267. E (Page 200)

268. D (Page 227)

269. A (Page 210)

419 ANSWERS

270. C (Page 35)

271. 8 (Pages 228, 229)

272. 0 (Page 213)

273. C (Pages 197, 200)

274. 0 (Page 201)

275. A (Page 201)

276. 0 (Page 87)

277. 8 (Page 227)

278. A - 6 8-9 C - 5 0-4 E- 10 Chapter 9 F-7 G - 8 H - 3 I - 1 J-2

279. 0 (Page 231)

280. 0 (Page 2(3)

281. 8 (Page 2(5)

282. A (Page 221)

283. E (Page 222)

284. C (Page 222)

285. E (Page 223)

420 ANSWERS

286. C (Page 213)

287. B (Page 213)

288. C (Page 214)

289. E (Page 217)

290. A (Page 250)

291. D (Page 218)

292. A - 2 (Pages 223-225) B-1 C-4 D - 3 E - 5

293. E (Page 213)

294. C (Page 204)

295. B (Page 232)

296. C (Page 232)

297. E (Page 233)

298. A (This page)

299. B (Page 223)

300. E (Page 223)

301. C (Page 195)

302. B (Pages 204, 205)

303. A (Page 212)

304. B (Page 143)

421 ANSWERS

305. E (pages 244, 245)

306. C (Page 235)

307. C (Page 249)

308. D (Pages 241, 242, 259)

309. D (Page 239)

310. E (This page)

311. E (Pages 258, this page)

312. A (Page 251)

313. B (Page 172)

314. E (Page 222)

315. B (Page 243)

316. D (This page)

317. B (Page 241)

318. B (Pages 243, 244)

319. A (Pages 258, 259)

320. D (Page 213)

321. E (Page 240)

322. E (Page 82)

323. C (Page 235)

324. F (Pages 75-78)

325. A (Page 79)

422 ANSWERS

326. E (Pages 270-272)

327. A (Page 274)

328. A (Page 1)

329. E (Page 262)

330. D (This page)

331. B (Pages 261, 262)

332. C (Page 262)

333. D (Page 264)

334. C (Page 268)

335. D (Page 266)

336. A (Page 265)

337. A (This page)

338. C (This page)

339. D (This page)

340. E (Page 270)

341. C (Page 270)

342. B (Page 267)

343. E (page 270)

344. C (page 271)

345. B (Page 272)

346. E (page 275)

423 ANSWERS

347. B (Page 274)

348. D (Page 275)

349. B (Page 293)

350. A (Page 293)

351. D (Page 293)

352. B (Pages 293, 294)

353. D (Pages 293, 294)

354. C (Page 293)

355. E (Page 297)

356. D (Page 296)

357. C (Page 296)

358. E (Page 293)

424